Exam 3 Things to Know, Exam 2 Things to Know, Hawkins BIO 111 Exam 1 Things to Know

¡Supera tus tareas y exámenes ahora con Quizwiz!

Diploid cell

- 2n - a cell containing 2 full sets of chromosomes

Cytokinesis

- division of the cytoplasm

If 2 cats with the genotypes, AAbbCc and aaBbcc, were mated, what percentage of their offspring would have the genotype, AaBbcc?

1 x 1/2 x 1/2 = 1/4

What is the hierarchical order of life?

1) Atom 2) Molecule 3) Cell 4) Tissue 5) Organ 6) Population 7) Community 8) Ecosystem

Jill wants a long haired, purple guinea pig. What is the probability of getting this guinea pig from parents that are blue, short haired and purple longhaired, assuming that the probability is not = to 0 (Blue color and short hair are dominant traits).

1/2 x 1/2 = 1/4

In cats, the homozygous ww have normal whiskers, Ww results in curly whiskers, and WW results in no whiskers. The genotype BB produces black fur, Bb gray fur, and bb white fur. If a cross is made between gray cats with curly whiskers and white cats with normal whiskers, what fraction of the offspring would be expected to have normal whiskers and gray fur?

25% Bbww

Jane's father is colorblind, but Jane and her husband have normal vision. What is the probability that Jane's first child will be a colorblind son? Assume this colorblind trait is X‐linked.

25% x^by

What are the results of meiosis?

4 unduplicated, haploid cells with 46 chromosomes

The fluid in the thylakoid of a chloroplast has 1000 times more H+ ions than the fluid in the stroma of the chloroplast. If the stroma fluid is pH 7.5, what is the pH of the thylakoid fluid?

4.5 7.5 - 3 = 4.5 When hydrogen goes up, the pH will go down, therefore substract 3 (because 10^3=1000).

Which of the following sequences is complementary to the DNA strand with sequence: 5' -TCCGAGCTA- 3'? A) 5' -TAGCTCGGA- 3' B) 5' -AGGCTCGAT- 3' C) 3' -ATCGAGCCT- 5' D) 3' -TCCGAGCTA- 5'

A) 5' -TAGCTCGGA- 3'

A carbon-12 atom has 6 protons and 6 neutrons. How many electrons does a carbon-12 atom have? A) 6 B) 4 C) 2 D) 12 E) Insufficient information has been given to answer his question.

A) 6

Which of the following is true about macromolecules? A) Are polymers made of smaller monomers. B) Always have a molecular weight greater than 100,000 da C) Are monomers made of smaller polymers D) Are always soluble in water E) Are not soluble in water

A) Are polymers made of smaller monomers Explain: Are polymers made of smaller monomers

Put the following events in the order they occur during the entire process of protein synthesis: I. Poly A tail addition II. transcription termination III. promoter binding IV. mRNA exits the nucleus V. tRNA binds the mRNA VI. RNA nucleotides added based off of template DNA strand A) III, VI, II, I, IV, V B) V, IV, I, II, IV, III C) III, IV, V, I, II, IV D) IV, III, I, II, V, VI E) None of the above

A) III, VI, II, I, IV, V

Unsaturated fatty acids __________. A) Increase the amount of HDL or "good" cholesterol. B) Are composed of straight chains of hydrocarbons. C) Increase the amount of LDL or "bad" cholesterol. D) Are more commonly found in animal fats as compared to plant fats. E) Are generally solid at room temperature.

A) Increase the amount of HDL or "good" cholesterol.

The first stage of translation and transcription is __________. A) Initiation B) Elongation C) Termination

A) Initiation

A pre-mRNA 5000 nucleotides long makes a protein consisting of approximately 500 amino acids. This result is best explained by which of the following? A) Introns are present in the pre-mRNA and are spliced out during pre-mRNA processing. B) There is redundancy in the genetic code. C) The RNA polymerase enzyme breaks off and re-attaches to the DNA strand at intervals. D) Several stretches of termination sequences occur within pre-mRNA. E) Ribosomes repeatedly pause and reinitiate translation after skipping several nucleotides.

A) Introns are present in the pre-mRNA and are spliced out during pre-mRNA processing.

Which of the following is true about the human genome? A) It's made up of 46 chromosomes. B) Only about 2% of our genome is considered non-coding. C) It is approximately 3 million basepairs (bp) long. D) Each pair of chromosomes are homologous, meaning they are identical. E) All of the above are true.

A) It's made up of 46 chromosomes.

In the generic redox reaction: A + B --> C + D, A) One substrate gains electrons while the other substrate loses electrons. B) A single substrate gains electrons. C) Both substrates and both products gain electrons. D) A single substrate loses electrons. E) Both substrates gain electrons and both products lose electrons.

A) One substrate gains electrons while the other substrate loses electrons.

When functioning properly, living organisms are _____________. A) Open systems B) Closed systems C) At metabolic equilibrium D) Defined by having delta G = 0 E) Always creating and destroying energy.

A) Open systems

Which property of genetic code lowers the chances of a deleterious mutation in an individual? A) Redundancy B) Universality C) Hydrogen bonding D) Uracil instead of thymine E) All of them

A) Redundancy

Which of the following statements is a reasonable explanation of why unsaturated fatty acids help keep a membrane more fluid? A) The double bonds form kinks in the fatty acid chain, preventing adjacent lipids from packing tightly. B) The double bonds block interaction among the hydrophilic heads of the lipids. C) Unsaturated fatty acids are more nonpolar than saturated fatty acids. D) Unsaturated fatty acids have a higher cholesterol content, which prevents adjacent lipids from packing tightly. E) They don't, saturated fatty acids keep the membrane more fluid because of their compact structure.

A) The double bonds form kinks in the fatty acid chain, preventing adjacent lipids from packing tightly.

What is unique about amphipathic lipids? A) They have a hydrophilic region. B) They increase your HDL or "good" cholesterol. C) They are all sterols (steroids). D) They can dissolve in water. E) They decrease your LDL or "bad" cholesterol.

A) They have a hydrophilic region.

What is the original source of electrons for PSII? A) Water B) Sunlight C) Molecular oxygen D) Carbon dioxide

A) Water

You have a stomach ulcer, a bacterial infection caused by H. pylori. Which of the following characteristics, if any, do you share with this pathogenic bacteria? A) You both have DNA composed of the nucleotides A, C, G, and T. B) You both have functioning mitochondria. C) You both belong to the same domain of life. D) You both are composed of millions of cells. E) You and H. pylori have nothing in common.

A) You both have DNA composed of the nucleotides A, C, G, and T.

The anticodon of a particular tRNA molecule is: A) complementary to the corresponding mRNA codon. B) complementary to the corresponding triplet in rRNA. C) The part of tRNA that bonds to a specific amino acid. D) Catalytic, making the tRNA a ribozyme.

A) complementary to the corresponding mRNA codon.

A point mutation that changes a codon for an amino acid to a stop codon is called: A) nonsense mutation B) missense mutation C) frameshift mutation D) stop mutation

A) nonsense mutation

Which list ranks the molecules from least information to most information? A) nucleotide, gene, chromosome, genome B) nucleotide, chromosome, genome, gene C) genome, chromosome, gene, nucleotide D) chromosome, genome, gene, nucleotide E) genome, chromosome, nucleotide, gene

A) nucleotide, gene, chromosome, genome

NAD+ is the ______ form of NADH while NADH is the _____ form of NAD+. A) oxidized; reduced B) reduced; oxidized C) cation; anion D) substrate; product E) product; substrate

A) oxidized; reduced

The research I conducted for completion of my PhD was to investigate plants that grew exceptionally well on really poor quality soils. One hypothesis we tested was to determine if one gene influenced several of the observed phenotypic traits allowing these plants to survive. This type of inheritance is called: A) pleiotrophy B) incomplete dominance C) quantitative D) polygeneic E) epistasis

A) pleiotrophy

We know that viruses require a host cell to survive. The virus will insert its nucleic acid into its host to replicate. However, the host cell can only take so much before it explodes. We have a virus with a genome 8000 bp in length that divides every 30 minutes, and a host cell with a viral load max of 500,000 bp. How long after infection will the host cell explode? A) ~ 3 hours B) ~ 24 hours C) ~ 0.5 hours D) ~ 96 hours E) ~ 500 hours

A) ~ 3 hours

What is the genotype of a homozygous dominant?

AA

What is the genotype of a heterozygous?

Aa

Miotic Spindle

An apparatus of microtubules that controls chromosome movement during mitosis

When do sister chromatids separate?

Anaphase of mitosis

What is a root?

Ancestral lineage that gave rise to all members of this tree

Given the following RF information, what is the likely order of the genes L, I, S, B? B to S = 31% I to B = 13% S to L = 23%

B L I S

During transcription, if the template strand of DNA has the sequence 3' TCTAGGACT 5', what will the sequence of the transcribed RNA be? A) 5' AGATCCTGA 3' B) 5' AGAUCCUGA 3' C) 3' UCAGGAUCU 5' D) 3' AGAUCCUGA 5' E) 3' AGATCCTGA 5'

B) 5' AGAUCCUGA 3'

Which molecule is important in regulating cell membrane fluidity? A) Protein B) Cholesterol C) Glucose D) Phospholipid

B) Cholesterol

What is produced when yeast undergo fermentation? A) Sugar B) Ethanol C) Oxygen D) Light E) Lactic acid

B) Ethanol

A scientific theory: A) Many incorporate explanations involving processes other than physical phenomena. B) Is a carefully-formulated proposition, based on observations, and supported by an accumulation of experimental data. C) Is a well-thought-out potential explanation for a phenomenon. D) May have been found to be false under some particular set of circumstances. E) Is a hypothesis that has yet to be subjected to experimentation.

B) Is a carefully-formulated proposition, based on observations, and supported by an accumulation of experimental data.

According to the fluid mosaic model, a membrane ________ A) Is composed of a single layer of fluid phospholipids between two layers of hydrophilic proteins B) Is composed of a fluid bilayer of phospholipids with embedded amphipathic proteins C) Is composed of a mosaic of fluid polysaccharides and amphipathic proteins D) Is composed of a fluid bilayer of phospholipids between two layers of hydrophilic proteins.

B) Is composed of a fluid bilayer of phospholipids with embedded amphipathic proteins

What happens to the oxygen that is part of the water molecules used in photosynthesis? A) Most is used in the light dependent reaction B) Most is released as waste C) Most is used to make glucose D) Most is used to make ATP E) Most is used to make NADPH

B) Most is released as waste

Which of the following is NOT an example of facilitated diffusion? A) Movement via ion channels B) Movement via an electrogenic pump C) Movement via water channel proteins D) All are examples of facilitated diffusion

B) Movement via an electrogenic pump

Which of the following is NOT a required function that all living organisms must accomplish? A) Maintaining homeostasis B) Natural selection C) Response to stimuli D) Growth and development E) Energy processing

B) Natural selection

What are the monomers of DNA and RNA? A) Triglycerides B) Nucleotides C) Monosaccharides D) Amino acids E) DNA/RNA are not composed of monomers.

B) Nucleotides

Which of the following is true about the citric acid cycle? A) It requires pyruvate as an input B) Only eukaryotic organisms can perform this process C) It requires oxygen D) It does not release/generate any CO2 E) It generates the most ATP among all respiration processes

B) Only eukaryotic organisms can perform this process

Theodor Engelmann illuminated a filament of algae with light that passed through a prism, thus exposing different segments of algae to different wavelengths of life. He added aerobic bacteria and then recorded which areas the bacteria congregated. He noted that the largest groups were found in the areas illuminated by the red and blue light. If you ran the same experiment without passing light through a prism, what would you predict to happen? A) The number of bacteria present would decrease due to an increase in the carbon dioxide concentration. B) The bacteria would be evenly distributed along the algae filaments. C) The bacteria would not be present anywhere along the filaments. D) The number of bacteria present would increase due to an increase in the carbon dioxide concentration. E) There would be no different in the result.

B) The bacteria would be evenly distributed along the algae filaments.

Which of the following distinguishes a eukaryotic cell from a prokaryotic cell? A) The number of cells present B) The presence or absence of internal membranes partitioning the cell C) The presence or absence of double-stranded DNA D) Whether or not ribosomes are present

B) The presence or absence of internal membranes partitioning the cell

What part of the cellular respiration pathway is affected by cyanide? A) The breakdown of sugars in glycolysis B) The transfer of electrons in the ETC C) The production of NADH in the Krebs cycle D) In ability of oxygen to enter the mitochondria

B) The transfer of electrons in the ETC

A black guinea pig crossed with a white guinea pig produced 12 black offspring. When the white pig was crossed again, using one of those 12 new black offspring, six black and six white pigs were obtained. What is the best explanation for this genetic situation? A) White is a dominant trait; black is incompletely dominant. B) White is a recessive trait; black is a dominant trait. C) White is a dominant trait; black is a recessive trait. D) White is a recessive trait; black is codominant. E) White and black are codominant.

B) White is a recessive trait; black is a dominant trait.

During photosynthesis, ___________ is oxidized and ________ is reduced. A) water; oxygen B) water; carbon dioxide C) water; glucose D) carbon dioxide; water E) carbon dioxide; glucose

B) water; carbon dioxide

Linked genes tend to ______________________. Which of Mendel's laws does this violate?

Be inherited together and are physically located near each other on the same chromosome. This violates MENDEL'S LAW OF INDEPENDENT ASSORTMENT.

Put the following events in the order they occur during the initiation stage of eukaryotic translation: 1. mRNA enters the cytoplasm and binds to small ribosomal subunit 2. A tRNA, carrying the amino acid MET, binds to the start codon on the mRNA. 3. Another tRNA binds in the A site in the ribosome. 4. Large ribosomal subunit binds to complex. A) 1, 4, 3, 2 B) 1, 3, 2, 4 C) 1, 4, 2, 3 D) 1, 2, 4, 3 E) None of the above

C) 1, 4, 2, 3 D) 1, 2, 4, 3

Cells in a plant root are non-photosynthetic, but they still depend on the light energy harvested during photosynthesis in leaves. In what form is the light energy transferred to the root cells? A) Electrons from the oxidation of CO2. B) Electrons from the oxidation of water C) Chemical energy stored in newly synthesized sugars D) Chemical energy stored in newly synthesized ATP

C) Chemical energy stored in newly synthesized sugars.

What is/are the end product(s) of the Calvin Cycle? A) NADPH B) All of these C) G3P D) Glucose

C) G3P

Even though plants carry on photosynthesis, plant cells still use their mitochondria for oxidation of pyruvate. When and where will this occur? A) In cells that are storing glucose only B) In photosynthetic cells in the light, while photosynthesis occurs concurrently C) In all cells all the time D) In nonphotosynthesizing cells only E) In photosynthesizing cells in the light and in other tissues in the dark.

C) In all cells all the time

If a cell has completed meiosis I and the first cytokinesis, and is just beginning meiosis II, which of the following is an appropriate description of its genetic contents? A) It has the same quantity of DNA and chromosomes as the parent cell. B) It is still diploid, but now the DNA is unduplicated (no sister chromatids). C) It is now haploid, but the DNA is still duplicated (present as sister chromatids). D) It has the same quantity of DNA as the parent cell, but now has variation from crossing over. E) It is still genetically identical to the other cell formed from the same meiosis I event.

C) It is now haploid, but the DNA is still duplicated (present as sister chromatids).

The energy source for photosynthesis is A) ATP B) CO2 C) Light D) Glucose E) Heat

C) Light

When sodium and chlorine combine to make sodium chloride, they create a(n) ______ bond, and the chlorine becomes a(n) _______ while the sodium becomes a(n) ________. A) polar covalent, anion, cation B) ionic, cation, anion C) ionic, anion, cation D) polar covalent, cation, anion E) non-polar covalent, cation, anion

C) ionic, anion, cation

Pinocytosis

Cell continually takes in droplets of extracellular fluid into vesicles

What type of molecules will require the most help to pass through a cell membrane? A) Small and nonpolar B) Small and polar C) Large and nonpolar D) Large and polar

D) Large and polar

Ribosomes are composed of ________ and ______ and directly synthesize ________________. A) Introns and exons; proteins B) mRNA and tRNA; polypeptides C) DNA and proteins ; polypeptides D) RNA and proteins ; polypeptides E) RNA and DNA ; proteins

D) RNA and proteins ; polypeptides

If you discovered a new organism and noticed that it uses anaerobic respiration and, among other things, produces CO2 as a byproduct, what assumption could you make? A) This organism is most likely NOT producing ethanol. B) This organism is most likely a yeast doing oxidative phosphorylation. C) This organism is most likely a bacteria that requires O2. D) This organism is most likely a yeast doing alcohol fermentation. E) This organism is most likely a yeast doing lactic acid fermentation.

D) This organism is most likely a yeast doing alcohol fermentation.

During photosystem II (PSII), oxygen is released as a waste product. What is the initial source of this oxygen? A) Photosystem I (PSI) B) Complex sugars (e.g. startch) C) Carbon fixation D) Water E) Atmospheric O2

D) Water

Alleles

Different versions of the same gene

Plutonium is a radioactive chemical element. It consists of 94 protons, 150 neutrons, and 94 electrons. What is its atomic mass (approx.) and atomic number, respectively? A) 94, 244 B) 94, 150 C) 150, 94 D) 94, 94 E) 244, 94

E) 244, 94 Explanation: Atomic # = # of protons --> in this case, there are 94 protons. Atomic mass = protons + neurons --> 94 + 150 = 244

Which of the following organisms perform aerobic cellular respiration? A) Animals B) Plants C) Fungi D) Bacteria E) All of the above

E) All of the above

A forensic scientist has isolated a sample from a corpse found at a remote crime scene in the Mojave Desert. This bizarre organism/sample is green in color and upon a quick glance under a microscope has linear chromosomes (linear genome), a flagella, lysosomes, ribosomes, and mitochondria. Given this information, this cell could be: A) A bacteria cell but not a plant cell. B) A bacteria cell or a plant cell. C) An animal cell or a bacterial cell but not a plant cell. D) A plant cell but neither a bacteria cell nor an animal cell. E) An animal cell but not a plant cell nor a bacteria cell.

E) An animal cell but not a plant cell nor a bacteria cell.

Which of the following structures is only found in animal cells when compared to plant cells? A) Cell walls B) Chloroplasts C) Mitochondria D) Golgi apparatus E) Centrioles

E) Centrioles

Which of the following statements is correct regarding entropy at the molecular level? A) Entropy of a gaseous state is less than the entropy of a liquid state. B) Entropy decreases when a substance is broken down into parts. C) Ice has more entropy than water molecules. D) A completed puzzle has more entropy than the unassembled puzzle pieces. E) Entropy increases as temperature increases.

E) Entropy increases as temperature increases.

Which of the following is a major difference between prokaryotic and eukaryotic protein synthesis? A) Eukaryotes usually employ the use of operons, or clusters of genes under the control of a single promoter. B) Eukaryotes use a much more advanced genetic code than prokaryotes. C) Prokaryotes use Okazaki fragments in the processing of mRNA to protein. D) Prokaryotes only do the process of transcription, translation is not needed. E) Eukaryotic mRNA must be processed with a 5' cap, 3' tail, and intron splicing.

E) Eukaryotic mRNA must be processed with a 5' cap, 3' tail, and intron splicing.

Gray seed color in peas is dominant to white. Assume that Mendel conducted a series of experiments where plants with gray seeds were crossed among themselves, and the following progeny were produced: 302 gray and 98 white. What is the most probable genotype of each parent? A) GG × gg B) GG × Gg C) gg × gg D) gg × Gg E) Gg × Gg

E) Gg × Gg

9) Why does Norman Borlaug have a building on campus named after him? A) He discovered and mapped out the Calvin Cycle (aka Calvin-Borlaug Cycle). B) He opened and continues to run a seed vault housing over 70,000 crop lines in a "bank" built into a mountain in the Arctic circle. C) He made the connection that wavelength greatly determined the efficiency of photosyntheis. D) He incorrectly hypothesized that plants were made out of water. E) He is a Nobel Prize winning plant biologist credited with saving over a billion people from starvation.

E) He is a Nobel Prize winning plant biologist credited with saving over a billion people from starvation.

Which of the following is an example of kinetic rather than potential energy? A) Water that is behind a dam B) The energy in an electrical field. C) Glucose (energy stored in chemical bonds). D) A car parked at the top of a hill. E) Heat evaporating from the skin.

E) Heat evaporating from the skin.

This is the summary equation for the process of photosynthesis, which is best described as a(n) _______________ reaction. A) double displacement B) catabolic C) spontaneous D) exergonic E) endergonic

E) endergonic

Which molecule is common output of glycolysis, pyruvate oxidation, AND the citric acid cycle? A) ADP B) Pyruvate C) Glucose D) NAD+ E) O2 F) NADH

F) NADH

What type of change would cause a frameshift mutation? A) deletion of 12 nucleotides B) substitution of G instead of "normal" T C) insertion of 12 nucleotides D) duplication of 13 amino acids E) all of these would cause a frameshift mutation F) none of these would cause a frameshift mutation

F) none of these would cause a frameshift mutation

What is nondisjunction?

Failure for chromosomes to separate properly.

A photosynthetic plant cell does not need mitochondria. True or false?

False

What is oogenesis?

Generation of egg scells

Diploid

Get two copies of each chromosome: one from mom and one from dad

Who set the foundation for genetics, and what did he study?

Gregor Mendel in the mid 1800s studied pea plants.

Haploid

Having a single set of unpaired chromosomes (n)

What is the oldest and most frequently used immortal cell line?

HeLa cell line

Why are males more prone to x-linked diseases?

In any XX/XY system, males are always more prone to x-linked diseases. They have no chance of "hiding" behind being a carrier.

A field full of weeds will compete for which resources required for photosynthesis? A) H20, O2, nitrogen B) Light, CO2, H2O C) NADP+, CO2, O2 D) Nitrogen, NADP+, CO2

Light, CO2, H2O

Branches

Lines uniting nodes, sometimes called edges

What is polytomy?

More than two lineages diverged and we don't know exactly how

When an error occurs in replication, it changes the nucleotide sequence of the DNA. This phenomenon is called a _______.

Mutation

In the photosynthesis experiment, what is the negative control?

No light --> complete darkness

Recombination

Offspring that show new combinations of the parental traits

Where does meiosis occur?

Ovaries and testes to produce haploid cells

When does crossing over occur?

Prophase of meiosis I

In Mendel's experiment, which flower color was dominant?

Purple

A homozygous tomato plant with red fruit and yellow flowers was crossed with a homozygous tomato plant with golden fruit and white flowers. The F1 all had red fruit and yellow flowers. The F1 were testcrossed by crossing them to homozygous recessive individuals, and the following offspring were obtained (below). How many map units separate these genes? (Remember, map units = RF) Red fruit and yellow flowers ‐ 41 Red fruit and white flowers ‐ 7 Golden fruit and yellow flowers ‐ 8 Golden fruit and white flowers ‐ 44

Recombinant = 15 8 + 7 = 15

Between which two genes would you expect the highest frequency of recombination? And why is the answer not G - B?

S - B = 40% It cannot be G-B because it is >50.

What are the results of meiosis II?

Sister chromatids are separated into daughter chromosomes. - This results in 4 UNDUPLICATED, HAPLOID DAUGHTER CELLS

Locus

Specific locations on a chromosome where alleles are found

Receptor-mediated endocytosis

Taking in cholesterol for membrane synthesis in humans

How do we identify interphase?

The DNA is in the form of chromatin. They are not visible (yet).

In preparation for cell division, what do the DNA and the chromosomes do?

The DNA is replicated, and the chromosomes condense

What are the results of Morgan's experiment?

The F2 generation showed a typical Mendelian 3:1 ratio of red eyes to white eyes. However, no females displayed the white-eye trait; they all had red eyes. Half the males had white eyes and half had red eyes.

You are testing the effect of pH on coral bleaching. What is your DEPENDENT variable in this experiment? A) The amount of coral bleaching. B) The type of coral used. C) The amount of time that the coral was allowed to grow. D) The pH of the water used. E) The presence or absence of water.

The amount of coral bleaching.

What are genes?

The basic units of inheritance and are comprised of DNA

Nodes

The branching points

What are the results of meiosis I?

The duplicated, homologous chromosomes, consisting of sister chromatids, are separated. - This results in DUPLICATED, HAPLOID CHROMOSOMES (2n --> n)

What is crossing over?

The exchange of genetic material by homologous chromosomes

Using Recombination Data

The farther apart genes are on a chromosome, the more likely it is for a crossover event to occur between them.

Genotype

The genetic makeup of an organism

What is basal taxon?

The oldest species, most ancestral

Phenotype

The physical appearance of an organism

What are cells?

The subunits of life

What is the disadvantage of asexual reproduction?

There is no genetic variation

In preparation for cell division, how are the chromosomes represented?

They are represented as sister chromatids.

Similarities between prokaryotes and eukaryotes

They both have: - cell membranes - cytoplasm - ribosome - DNA

Through the Chromosomal Theory of Inheritance, what did Sutton & Boveri demonstrate?

They demonstrated that chromosomes occur in matched maternal & paternal pairs that segregated during meiosis.

According to Darwin's Theory, who are the most fit individuals?

Those who leave the most fertile offspring

Laws of Probability

Used to predict most likely genotypes of offspring

Phagocytosis

Vesicle which pinches off does not have a coated pit

What does phylogeny set out to answer?

What species did an organism evolve from? What species is an organism most closely related to?

In Mendel's experiment, which flower color was recessive?

White

In the photosynthesis experiment, what is the positive control?

White sunlight

What happens within cells?

Within these cells, the necessary reactions for life occur

Lysosome

breakdown of ingested macromolecules and other substances

Mitochondria

convert energy to forms that cells can use for work

Peroxisome

metabolic compartment enclosed by a single membrane

You decided to grow some peppers plants: one of those plants yielded peppers that were mild and green, while the other produced fruits that were spicy and red. You then wanted to know whether the genes responsible for these traits were linked, so you crossed your true‐breeding plants and got an F1 generation that produced peppers that were mild tasting and green. You then took those F1 plants and crossed them back to your original plant that produced spicy red peppers. The offspring of that cross and the types of peppers they produce are listed below. 425 Mild, green; 375 Red spicy; 125 Mild, red; 75 Green spicy What is the number of offspring that are "parental" types? What is the number of offspring that are "recombinant types"? What is recombination frequency for these two traits?

n = 1000 - Parental = 425 + 375 = 800 - Recombinant = 125+75 = 200 200/1000= 20%

You found a new species of starfish that vary for color and number of arms. You cross two "pure bred" parents together to get an F1 hybrid population that were all orange and had 7 arms. To determine whether these traits were linked, you did a testcross with the F1 starfish by breeding them to a true‐breeding blue, 5 armed starfish. The resulting F2 offspring are listed below. 275 Orange, 7 arms; 235 Blue, 5 arms; 60 Orange 5 arms; 30 Blue, 7 arms What is the number of offspring that are "parental" types? What is the number of offspring that are "recombinant types"? What is recombination frequency for these two traits?

n = 600 Parental = 275+235 = 510 Recombinant = 60+30 = 90 90/600 = 15%

Nucleus

physically separate the DNA from most metabolic processes

Golgi apparatus

protein, glycoprotein, and phospolipid modification

What is the phenotype of a heterozygous?

purple

Vacuole

storage and waste disposal

During photosynthesis, light-independent reactions take place in the ___________ and the light reactions take place in the _________.

stroma; thylakoid

What is phylogeny?

the evolutionary history and relationship of an organism and groups of organisms.

What is the phenotype of a homozygous recessive?

white

Electronegativity

- # of protons in the nucleus (more protons = more electronegative) - # of electrons (more electrons = more electronegative) - The distance of the outer electrons from the nucleus (greater distance = less electronegative)

Codominance

- 2 dominant alleles affect the phenotype in separate, distinguishable ways - Example: human blood type - When two alleles express separate and equal phenotypic traits - The heterozygous individual express both traits

Pleiotropy

- A gene has multiple phenotypic effects - Examples: Sickle Cell Disease and Mendel's Pea Plants

Chemical Groups

- Added on to hydrocarbons, simplest organic molecules, creating more complex organic molecules - Replace the hydrogens in a hydrocarbon - Nearly all of the molecules found in living organisms aren't just hydrocarbons - Small changes can have a large impact

What are the conclusions of Morgan's experiment?

- All F1 offspring had red eyes, so the mutant white eye trait must be recessive to the wt red-eye trait. - Since the recessive trait was expressed only in males in the F2 generation, the eye-color gene must be located on the X chromosome.

Cell Theory Tenets

- All living organisms are made of one or more cells. - Chemical reactions necessary for life take place within cells. - All cells arise only from preexisting cells. - Cells contain hereditary information in the form of DNA.

Law of Independent Assortment

- Alleles of unlinked genes assort independently and are randomly distributed into gametes - Mendel identified his 2nd law of inheritance by following 2 characteristics - This produced dihybrids in the F1 generation: heterozygous for both characteristics

Law of Segregation

- Alleles will segregate during meiosis and gametes have an equal change of receiving either allele. - Homologous chromosomes are separated from each other.

Thalidomine

- Both a sedative and teratogenic - Was given to women in the late 50s for nausea/insomnia during pregnancy - Banned after causing 1000s of deaths - Can administer the good enantiomer, but it is converted by the body to the harmful one - Reapproved in late 90s to treat leprosy and certain cancers.

What is evolution?

- Change over time - Descent with modification - Change in allele frequency - Such changes can lead to new species

Prophase I of Meiosis I

- Chromatin condenses - Pairing of homologous chromosomes - Crossing over occurs - Replicated centrioles move to poles - Nuclear membrane disintegrates

Binary Fission

- Circular DNA replicated - Replicated strands attach to the plasma membrane - Plasma membrane elongates, separating the DNA - Plasma membrane grows inward at the center of the parent cell - This then separates into 2 daughter cells - The daughter cells are identical

Isomers

- Compounds that have the same number and types of atoms of an element, but they are arranged differently.

What are the major differences between meiosis I vs mitosis?

- Crossing over during prophase I - Alignment of homologous pairs at the metaphase plate - Separation of homologous chromosomes

S phase of interphase

- DNA replicates - Chromosomes duplicate to form sister chromatids

Inductive reasoning

- Describes natural structures and processes as accurately as possible through careful observation and analysis of data - Scientists derive generalizations based on a large number of specific observations - Example: The sun will rise tomorrow --> many observations have led us to infer this conclusion about the sun.

Structural aspects of isomers

- Differ in covalent arrangement - Same molecular formula: C5H12 - Hydrocarbons not found in organisms - Pentane straight, 2 methyl branched - Pentane found in gas, used to make styrofoam - 2-methylbutane predominant in gasoline

Geometric aspects of isomers

- Different in spatial arrangement around double bonds - Vision based on a molecule being converted from cis to trans

What is an allele, and what is its significance?

- Different versions of the same gene - They exist and cause variation

Carbonyl group

- Double bonded molecules of O and C - 2 types found in carbs: - Aldehydes: terminal - Ketones: internal - One or the other on every monosaccharide

Nature vs. Nurture

- Environmental impact on phenotype is HUGE and dynamic

Who is Gregor Mendel?

- Father of modern genetics - Set the principles of genetics by documenting a particulate mechanism of inheritance - Discovered laws that govern the passage of traits from one generation to the next - He manually cross-pollinated plants.

Turner's Syndrome

- Female, often infertile - Significant issues related to heart and kidneys

Hydroxyl group

- Found in alcohols and sugars - Polar - due to electronegative oxygen, increases the solubility of molecules - O -H

Hydrogen bonds between functional groups

- Functional groups that help organic compounds dissolve in water -- POLAR - If they are polar, they can form hydrogen bonds - Hydrogen bonding between functional groups is extremely important for biological molecules

The Gene Theory

- Genes are the most basic units of inheritance and are comprised of DNA - Provides the foundation for the field of genetics

Enantiomers

- Have four different groups bonded to an asymmetric carbon; mirror images - You cannot superimpose the two structures on one another - Very important in pharmaceutical industry - One _____ will be effective, the other will not

Who is Thomas Hunt Morgan?

- He used fruit flies as a model organism. - First observed and noted "wild type"

What are the names of the three genotypes?

- Homozygous dominant - Homozygous recessive - Heterozygous

Amino Group

- In every amino acid - Nitrogen atom attached to two hydrogen atoms

Carboxyl group

- In fatty acids AND amino acids - Acidic properties - Double bond: carbon and oxygen - Single bond: carbon and hydroxyl - Written as COOH and CO2H

Deductive reasoning

- Inquiry that asks specific questions and tests a hypothesis - The logic flows from the general to the specific - If a hypothesis can be supported then we can expect a particular outcome.

How does science differ from inquiry?

- It depends on observations and measurements that others can repeat and verify. - It requires hypotheses and theories to be testable by observations and experiments that can be repeated. - It requires hypotheses and theories to be falsifiable and impossible to prove. - Beliefs, opinions, and the supernatural are not falsifiable and therefore fall out of the realm of science.

Emergent Properties of Life and Levels of Organization

- Life's fundamental characteristic is having a high degree of order - Biological organization is based on hierarchy of structural levels - These properties result from interactions between components - These properties reflect a hierarchy of structural organization and regulation.

Metaphase

- Longest stage of the cell cycle - Chromosomes are aligned along the equatorial plane - DNA is still duplicated - Chromosomes are represented as sister chromatids

Why do cells need to reproduce?

- Make new organisms - Repair damage - Growth - Tissue renewal - Replicates their DNA

Jacob's Syndrome

- Male (XXY) - Symptoms include: attention difficulties, delayed motor skills and speech, involuntary muscle movement, and emotional or behavioral issues

Klinefelter's Syndrome

- Male (XXY), usually infertile - Most common cause of hypogonadism, a condition that stops the body from producing sperm and testosterone

Quantitative Inheritance

- Many human characteristics vary in population along a continuum - Quantitative variation usually indicates polygenic inheritance

What did Thomas Hunt Morgan do in his experiment?

- Mated male flies with WHITE EYES with female flies with RED EYES -- WT

Cytokinesis in animal cells

- Microtubules help form a furrow that constricts cytoplasmic mass into 2 daughter cells - Cleavage furrow forms from the outside and moves inward

Dopamine

- Molecular structure: benzene structure - Some people have too much and need to take meds to lower levels - Some people have too little and need to take meds to increase levels

Trisomy 21

- Most common genetic birth defect - Range of traits vary from mild to severe (increased risk of leukemia, generally vary social and outgoing, issues with immune, GI, and nervous systems, characteristics facial and limb features)

Cytokinesis in plant cells

- No cleavage furrow - Rather, vesicles form a cell plate that forms from the middle and moves outward

Telophase

- Nuclear envelope forms around each sets of chromosomes - Chromosomes de-condense - Nuclei form for each daughter cell - DNA is unduplicated - No longer sister chromatids

Telophase I of Meiosis I

- Nuclear envelope reforms around separated chromosomes - Cytokinesis may follow telophase I, forming two haploid daughter cells - Sister chromatids may not be identical due to crossing over

Prophase

- Nuclear membrane breaks down - Chromosomes continue to condense until visible - DNA is still duplicated - Chromosomes are represented as sister chromatids

Complete Dominance

- Occurs when the phenotypes of the heterozygous and dominate homozygote are identical - Like purple pea flowers

Epistasis

- Phenomenon where the effect of a gene depends on the presence or absence of one or more other genes - One gene affects the phenotype of another gene - Example: red heads

G2 of interphase

- Plants -- spindle assembly forms - Animals -- centrioles divide - Chromosomes begin to condense (become compact) - Now ready for mitosis

G1 phase of interphase

- Primary growth period - Cell doubles in size - Organelles duplicate

Asexual Reproduction

- Produces identical offspring - Generation of offspring from a single individual - Offspring genetically identical to parent and siblings

Sexual reproduction

- Produces variable offspring - Allows organisms to begin combing genes, allowing the next generation to do more than its parents, which ups the chance of survival - Creates complexity and ways to survive a changing world - The variation is produced by behavior of chromosomes during sexual life cycle.

Basic science

- Science done to simply gain knowledge, regardless of application - No product or service going in - Knowledge for the sake of knowledge

Anaphase I of Meiosis I

- Separate homologous chromosomes to each poles - Sister chromatids remain attached

Anaphase

- Shortest stage of the cell cycle - Centromeres divide, separating the sister chromatids - Each is now pulled toward the pole to which it is attached by spindle fibers - Chromatids begin to separate

Polygenic

- Some characteristics/traits may be determined by 2 or more genes - Epistasis

Applied science

- Technology - Science applied to real world problems - Problem is defined by each researcher

Metaphase I of Meiosis I

- The chromosomes unwind - The spindle fibers attach chromosomes at kinetochore to centrosomes. - Align chromosomes at metaphase plate, with each homologous pair on either side of the plate

Functional group

- The components of organic molecules that are most commonly involved in chemical reactions - Behaves consistently from one organic molecule to another - Provide a way to differentiate between differentiate bio molecules - Increase the solubility of organic compounds in water

Incomplete Dominance

- The phenotypes of F1 hybrids are somewhere between the phenotypes of the 2 parental varieties - 3 phenotypes are seen usually - F1 hybrids exhibit a phenotype that falls somewhere in between the two parental varieties - A blending of traits - Red + white = pink

What are the advantages of asexual reproduction?

- The population can increase rapidly when the conditions are favorable. - Only one parent is needed - It is more time and energy efficient as you don't need to find and maintain a mate. - It is faster than sexual reproduction

Why did Morgan use fruit flies?

- They breed at a high rate. - A new generation can be bred every 2 weeks. - They have only 4 pairs of chromosomes. - Super easy to deal with in a lab setting.

Karyokinesis

- division of the nucleus

Eukaryotes

- large and complex - unicellular or multicellular - has nucleus - linear DNA - diploid (2n) chromosome - has membrane-bound organelles - mostly reproduce sexually - cell division by mitosis - plant and animal cells

Haploid cell

- n - a cell containing 1 full set of chromosomes

You are doing research on snapping turtles and are trying to determine the recombination frequency of two genes that control shell texture and overall color. To achieve this you bred the 2 true breeding individuals shown above together to produce an F1 group that are all dark in color and have spiny shells. A couple months ago you bred the F1 individuals to an albino smoothed shell turtle and today the eggs hatched! When you counted the babies, you found the following results: 100 Dark/spiny shells; 80 Albino/smooth shells; 15 Dark/smooth shells; 5 Albino/spiny shells What is the number of offspring that are "parental" types? What is the number of offspring that are "recombinant types"? What is recombination frequency for these two traits?

- n = 100 + 80 + 15 + 5 = 200 - Parental = 100 + 80 = 180 - Recombinant = 15 + 5 = 20 20/200 = 10%

What is biology?

- the study of life - the field of science that is related to the physical world and its phenomena and processes

Prokaryotes

- unicellular - no nucleus - circular DNA - single haploid (n) chromosome - lack membrane bound organelles - reproduce both sexually and asexually - cell division by binary fission - bacteria

Methyl Group

- unreactive - contributes to the shape of many organic molecules

Calculate the probability of getting the genotype AabbCCdd x AabbCcDd

1/2 x 1/2 x 1/2 x 1/4 = 1/32

What is the [OH-] in a solution with a pH of 3? A) None of the above. B) 10^11 C) 10^3 D) 10^-11 E) 10^-3

10^-11

What is the result of mitosis?

2 genetically identical daughter cells

What are the results of interphase in meiosis I?

2 sister chromatids attached at centromeres

How much NaCl would you need to make a 0.100 L (100 mL) of a 0.5 molar solution? NaCl has a molecular weight of 58.44 g.

2.9 g

How long does the human cell cycle last?

24 hours

How many chromosomes do humans have?

46 chromosomes, 23 pairs

Arnold has a sister with cystic fibrosis (CF), an autosomal (not sex‐linked) recessive disease. None of Arnold's grandparents have ever been diagnosed with CF. What is the probability that Arnold is a carrier (heterozygous) of a CF disease allele?

50% because Arnold's sister had to inherit one recessive allele from each parent, so they must be Ff x Ff

Drosophila melanogaster, the common fruit fly, has 4 chromosomes in each of its gametes. How many chromosomes are in each somatic cell?

8

You want 100 mL of 2 M NaOH. How many grams NaOH do you add to enough water to make 100 mL? The molar mass of 40 g.

8 grams Explanation: 2 M is 2 mol/L. Since you want only 100 mL (=0.1 L) you need 0.2 mol. At 40 g/mol, this is 8 g.

Triple X Syndrome

A genetic abnormality in which a female has an extra X sex chromosome; characterized by decreased fertility, some genital abnormality, and slight mental retardation.

Character

A heritable feature

Trait

A variant of a character

In a tetrahybrid cross (4 genes of interest) two pure bred parents mate (WWXXYYZZ x wwxxyyzz) to produce an F1 hybrid generation. One F1 progeny is used in a test cross (aka back-crossed) with the pure bred tetra-mutant parent. What frequency of F2 progeny have the genotype WwxxYyZZ? A) 0 B) 1/256 C) 1/128 D) 1/64 E) 100

A) 0

A couple finds out that they are having a baby. The dad-to-be has Duchenne Muscular Dystrophy (a sex-linked recessive condition) and the mom-to-be is a carrier. What is the chance that a daughter will be born who is not affected (doesn't have the disease nor is she a carrier)? A) 0 (no chance) B) 33% C) 50% D) 75% E) 100% (certain)

A) 0 (no chance)

What do mitochondria and chloroplasts have in common? 1. Double membrane 2. Make sugar from air and light 3. Found in animal cells 4. Found in plant cells 5. Not part of endomembrane system A) 1, 4, 5 B) 1, 3, 4, 5 C) 1, 2, 3, 4, 5 D) 1, 2, 4 E) 2, 3, 4

A) 1, 4, 5

Carbon-14 decays to nitrogen-14 with a half-life of approximately 5700 years. After 57,000 years, approximately what fraction of the initial carbon-14 would be left in an organic sample? A) 1/1000 B) 1/2 C) 1/100 D) 1/20 E) 1/10

A) 1/1000

You discover a new organism and analyze its DNA. If you find 40% of nucleotides are G, which of the following is true: A) 10% are A B) 20% are A C) 10% are C D) 20% are C

A) 10% are A

In a particular species of mountain goats, the amount of purine nucleotides are equal, each making up 18% of the total nucleotide count. How many guanines are present in this goat's genome? A) 18% B) 32% C) 36% D) 64% E) Cannot determine with the information provided.

A) 18%

If an atom has 5 electrons in its outer shell, how many of these electrons are unpaired? A) 3 B) 5 C) 2 D) 1 E) None of these F) 8

A) 3

Arrange the events taking place during transcription in order: 1. The pre-RNA undergoes processing. 2. RNA polymerase moves downstream, unwinding the DNA. 3. RNA polymerase binds to the promoter. 4. RNA transcript is released and polymerase detaches from the DNA. 5. Polymerase initiates RNA synthesis at the start point on the template strand. A) 3, 5, 2, 4, 1 B) 5, 2, 1, 3, 4 C) 4, 3, 2, 1, 5 D) 1, 3, 5, 2, 4

A) 3, 5, 2, 4, 1

How many covalent bonds can a Carbon atom form? A) 4 B) 2 C) 0 D) 8 E) 6

A) 4

Given below is a list of events taking place during translation. Arrange these in proper sequence. 1. The anticodon of tRNA base pairs with the mRNA codon at the A site. 2. Peptide bond formation between amino group of amino acid in A site and the carboxyl end of the polypeptide in the P site. 3. Translocation of tRNA from the A site to the P site. 4. A small ribosomal subunit binds with mRNA. A) 4, 1, 2, 3 B) 4, 1, 3, 2 C) 1, 2, 4, 3 D) 2, 1, 4, 3

A) 4, 1, 2, 3

Based on your understanding of biological macromolecules, the number of unique monomer molecules that is common to living organisms is closest to which number? A) <100 B) 10,000 C) 1,000 D) >100,000

A) <100

In humans, ABO blood types refer to glycoproteins in the membranes of red blood cells. There are three alleles for this autosomal gene: IA, IB, and i. The IA allele codes for the A glycoprotein, The IB allele codes for the B glycoprotein, and the i allele doesn't code for any membrane glycoprotein. IA and IB are codominant, and i is recessive to both IA and IB. People with type A blood have the genotypes IAIA or IAi, people with type B blood are IBIB or IBi, people with type AB blood are IAIB, and people with type O blood are ii. If a woman with type AB blood marries a man with type O blood, which of the following blood types could their children possibly have? A) A and B B) A, B, AB, and O C) AB and O D) AB and O E) O

A) A and B

Which of the following is not common to both prokaryotic and eukaryotic cell division? A) A checkpoint will be activated if the spindle does not attach to a kinetochore. B) The DNA is replicated. C) Two genetically identical daughter cells are produced. D) Cleavage occurs to separate the cell in two.

A) A checkpoint will be activated if the spindle does not attach to a kinetochore.

Which of the following is most likely to have a ubiquitin "tag" and get sent to the proteosome for processing? A) A protein that has completed its job and now is no longer needed. B) A newly transcribed mRNA sequence before it leaves the nucleus. C) DNA that has just been wrapped around histones to form a nucleosome. D) A mature mRNA that just got into the cytoplasm after leaving the nucleus. E) The small ribosome subunit that attached itself to the mature mRNA.

A) A protein that has completed its job and now is no longer need

What differentiates a hypothesis from a theory? A) A theory is a principle that has been formed as an attempt to explain things that have already been substantiated by decades of data. B) A hypothesis is an attempt to explain an observation, a theory is a speculation or hunch about something. C) A hypothesis has to be able to be testable, but a theory does not. D) A theory can be proven, a hypothesis cannot. E) A hypothesis can be rejected, a theory cannot.

A) A theory is a principle that has been formed as an attempt to explain things that have already been substantiated by decades of data.

What is the source of energy during the energy investment phase of glycolysis? A) ATP B) Glucose C) Heat D) NAD+ E) NADH F) light

A) ATP

What major event happens during the light-independent (dark) reactions of photosynthesis? A) ATP and NADPH are used to convert molecules of 3-PGA into molecules of glyceraldehyde 3-phosphate (G3P) B) CO2 is fixed by glucose C) Energy from the sun is passed around photosystems until reaching the reaction centers of the cluster of pigments D) The energy in sugar is transformed immediately into ATP. E) RUBISCO breaks down water and releases high energy electrons.

A) ATP and NADPH are used to convert molecules of 3-PGA into molecules of glyceraldehyde 3-phosphate (G3P)

The role of NAD+ in cellular respiration is to: A) Accept electrons and pass them to the electron transport chain. B) Donate electrons and accept them from the electron transport chain. C) Reduce glucose to CO2. D) Oxidize glucose to CO2.

A) Accept electrons and pass them to the electron transport chain.

Acetylation of histones in the vicinity of a gene causes the gene to be A) Activated because the histones bind less tightly to the DNA than unacetylated histones B) Silenced because the histones bind less tightly to the DNA than unacetylated histones. C) Activated because the histones bind more tightly to the DNA than unacetylated histones. D) Silenced because the histones bind more tightly to the DNA than unacetylated histones.

A) Activated because the histones bind less tightly to the DNA than unacetylated histones

This is Theodor Engelmann's experiment utilizing algae, oxygen-dependent bacteria, and the spectrum of visible light. What major conclusion is observed here? A) Algae performs photosynthesis most effectively in the blue and red portions of the spectrum, therefore bacterial growth is optimal at those wavelengths as well. B) The bacteria grew best in the green portion of the spectrum. C) Bacteria performs photosynthesis most effectively in the blue and red portions of the spectrum, therefore algae growth is optimal at those wavelengths as well. D) The bacteria killed the algae, wavelength had no effect. E) The bacteria could not grow with the algae present, wavelength had no effect.

A) Algae performs photosynthesis most effectively in the blue and red portions of the spectrum, therefore bacterial growth is optimal at those wavelengths as well.

During cellular respiration, what happens to the 6 carbons in glucose? A) All 6 carbons are completely oxidized to CO2. B) 3 carbons are oxidized to CO2 and 3 carbons are reduced to CO2. C) All 6 carbons are reduced to CO2. D) All 6 carbons are used in the synthesis of ATP.

A) All 6 carbons are completely oxidized to CO2.

Which of the following is not a polymer? A) An amino acid molecule B) An RNA molecule C) A chitin molecule D) A DNA molecule E) A starch molecule

A) An amino acid molecule

At which stage of meiosis do the centromeres of sister chromatids disjoin and chromatids separate. A) Anaphase II B) Prophase II C) Prophase I D) Anaphase I E) This does not happen during any stage of meiosis.

A) Anaphase II

In which stage of polymerase chain reaction (PCR) are the specific primers added to the DNA strands? A) Annealing B) Denaturation C) Elongation D) Termination

A) Annealing

Which of the following is true about the human genome? A) Approximately 98% of our genome is considered non-coding or "junk" DNA. B) It is the most complex, therefore the largest, among all known species. C) It consists of a singular, long DNA molecule connected at the ends to form a circle. D) It's made up of 46 pairs of chromosomes. E) All of the above are true.

A) Approximately 98% of our genome is considered non-coding or "junk" DNA.

What emergent property of water accounts for the phenomenon seen above? (Paper towels between cups) A) Capillary action (cohesion/adhesion) B) Its ability to expand upon freezing. C) Its versatility as a solvent. D) Its low viscosity. E) The ability to moderate temperature.

A) Capillary action (cohesion/adhesion) Explanation: The paper towels move H2O against gravity.

Which of the following effects might be caused by reduced or very little active telomerase activity? A) Cells age and begin to lose function. B) Cells maintain normal functioning. C) Cells may become cancerous D) Telomere lengthens in germ cells

A) Cells age and begin to lose function.

Which of the following DOES NOT support the endosymbiotic theory? A) Chloroplasts and mitochondria have the same sizes. B) Chloroplasts and mitochondria are both double-membrane bound. C) Chloroplasts and mitochondria contain their own DNA. D) Chloroplasts and mitochondria contain bacteria-like ribosomes.

A) Chloroplasts and mitochondria have the same sizes.

If a gene responsible for the production of telomerase was mutated and led to reduced, or even no production, of the enzyme, what would be the most probable outcome? A) Chromosome shortening/cellular aging. B) A rapid increase in cell/DNA replication. C) A significant delay in cellular aging. D) One of several types of cancer. E) This wouldn't have much of an effect on the cell.

A) Chromosome shortening/cellular aging.

In taxonomic hierarchy, two organisms could be in the same ______ but DIFFERENT ________. A) Class; orders B) Genus; Families C) Species; Genera (plural for Genus) D) Phylum; Kingdoms E) Family; Classes

A) Class; orders

What are the three basic steps of conventional PCR? A) Denaturation, Annealing, Extension B) Denaturation, Annealing, Strand displacement C) Strand displacement, Synthesis, Release D) Synthesis, Annealing, Extension

A) Denaturation, Annealing, Extension

The basis for dividing the prokaryotes into two domains is: A) Differences in genome sequences. B) Differences in metabolic properties. C) None of the choices. D) Differences in the ability to survive in extreme conditions.

A) Differences in genome sequences

Natural selection is: A) Differential reproductive success of different phenotypes B) The natural "force" causing random changes in the gene pool C) The ability to survive and leave offspring D) Change over time E) A product of the genotype and environmental factors

A) Differential reproductive success of different phenotypes

Which of the following groups of membranes and/or organelles is considered part of the cell's endomembrane system? A) Endoplasmic reticulum, Golgi apparatus, nuclear envelope B) Golgi apparatus, chloroplasts, nuclear envelope C) Nuclear envelope, mitochondria, endoplasmic reticulum D) Lysosomes, vacuoles, peroxisomes E) Cell wall, lysosomes, Golgi apparatus

A) Endoplasmic reticulum, Golgi apparatus, nuclear envelope

What is the substrate for the first step in glycolysis? A) glucose B) pyruvate C) ATP D) carbon dioxide - CO2

A) Glucose

Which of these factors are required for biological evolution to occur? A) Heritable variation in the population B) Many generations C) A large population D) Natural selection E) Variation in acquired traits among individuals in the population.

A) Heritable variation in the population

The following scientist(s) demonstrated that DNA was the hereditary material by performing this famous experiment. A) Hershey and Chase, radio-labeling DNA and protein B) Watson and Crick, x-ray crystallography and DNA structure C) Hershey and Chase, transformation of DNA in Streptococcus pneumoniae D) Frederick Griffith, transformation of DNA in Streptococcus pneumoniae E) Watson, Crick, and Franklin, x-ray crystallography and DNA structure

A) Hershey and Chase, radio-labeling DNA and protein

What improves the productivity of cellular respiration? A) Highly folded surface of the inner mitochondrial membrane. B) The presence of free ribosomes in the mitochondrial matrix. C) The presence of multiple circular DNA molecules in the mitochondria. D) The presence of an intermembrane space.

A) Highly folded surface of the inner mitochondrial membrane.

Which of the following is FALSE about glycogen as a storage polysaccharide? A) Hydrolysis of glycogen releases fructose, a sugar that is sweeter than glucose and thus gives more stored energy. B) Glycogen stores are mainly replenished by food consumption. C) It is stored primarily in liver and muscle cells. D) It has a highly branched structure, which allows rapid mobilization of its sugar units during metabolic need.

A) Hydrolysis of glycogen releases fructose, a sugar that is sweeter than glucose and thus gives more stored energy.

Which of the following statements best describes the effect a nonsense mutation would have a gene? A) It introduces a premature stop codon into the mRNA. B) It has no effect on the amino acid sequence of the encoded protein C) It renders the gene completely useless/ineffective. D) It alters the reading frame of the mRNA. E) It changes an amino acid in the encoded protein.

A) It introduces a premature stop codon into the mRNA.

The most common form of Iron (Fe), atomic number 26, has an atomic mass of ~56. What is unique about the radioactive isotope 59Fe and how many neutrons does it have? A) It is unstable and has 33 neutrons. B) It is unstable and has 30 neutrons. C) It is unstable and has 26 neutrons. D) It is stable and has 33 neutrons. E) It is stable and has 26 neutrons.

A) It is unstable and has 33 neutrons. Explanation: Mass = p + n 59 = 26 + n n = 33

Which of the following class of biomolecules is NOT considered as a true polymer? A) Lipids B) Carbohydrates C) Nucleic acids D) Proteins

A) Lipids

At which stage of meiosis do the homologous chromosomes align at the equator of the spindle? A) Metaphase I B) Metaphase II C) Anaphase I D) Anaphase II E) This does not happen during any stage of meiosis.

A) Metaphase I

Can the light reactions of photosynthesis continue on indefinitely without the Calvin cycle? A) No, the light reactions require products of the Calvin Cycle to produce ATP and NADPH. B) Yes, the light reactions can produce ATP and NADPH without inputs from the Calvin cycle.

A) No, the light reactions require products of the Calvin Cycle to produce ATP and NADPH.

What type of covalent bonds exist in methane? A) Non-polar B) Ionic C) Hydrogen D) Polar

A) Non-polar

Which of the following basic molecular building blocks would be found in chromosomes? A) Nucleotides and amino acids B) Nucleotides only C) Nucleotides and carbohydrates D) Nucleotides and lipids

A) Nucleotides and amino acids

What type of molecule would require the least amount of help to pass through a cell's phospholipid bilayer? A) O2 (small, nonpolar molecule) B) Glucose (large polar molecule) C) DNA D) Na+ (ion) E) Starch (large insoluble polar molecule)

A) O2 (small, nonpolar molecule)

Scientists discovered an unusual liquid substance oozing out of a desert tree. After preliminary testing, the substance was determined to have a pH of 12. How many OH- and H+ ions does it have, respectively? A) OH = 10^-2; H = 10^-12 B) OH = 12; H = 2 C) OH = 12; H = 12 D) OH = 10^-12; H = 10^-2 E) OH = 10^2; H = 10^12

A) OH = 10^-2; H = 10^-12 Explanation: pH = 12 H+ = 10^-12 and OH = -2

We discussed a phenomenon called the Warburg Effect and its implications for glucose metabolism. Which of the following statements is true? A) Oxidative phosphorylation takes much more time to complete than anaerobic respiration. B) Aerobic respiration produces much less ATP than anaerobic respiration. C) Some cells, even in the absence of oxygen, will still manage to undergo aerobic respiration. D) Some cells, including certain cancer cells, are killed by this process. E) Anaerobic respiration is a much slower process than oxidative phosphorylation.

A) Oxidative phosphorylation takes much more time to complete than anaerobic respiration.

Which of the following is a major difference when comparing protein synthesis between prokaryotics and eukaryotes? A) Prokaryotes usually have operons, or clusters of genes under the control of a single promoter. B) Eukaryotes undergo the processes of transcription and translation concurrently. C) Prokaryotes use a polyadenylated sequence (AAUAAA) to signal the termination of transcription. D) Prokaryotic mRNA must be processed with a 5' cap, 3' tail, and intron splicing. E) Eukaryotes use a much more advanced genetic code than prokaryotes.

A) Prokaryotes usually have operons, or clusters of genes under the control of a single promoter.

Which of the following is NOT TRUE about RNA? A) RNA is exceptionally stable. B) RNA is involved in protein synthesis. C) RNA is single-stranded. D) RNA is found in the nucleus and in the cytoplasm. E) RNA is made up of four nucleotides.

A) RNA is exceptionally stable.

The graph best depicts the relationship between glucose levels and ATP production by a cell in an otherwise closed system. Explain why. A) Since nothing new can enter or leave the system, there is a negative relationship between glucose levels and ATP concentration. B) Since nothing new can enter or leave the system, there is a positive relationship between glucose levels and ATP concentration. C) Because in a closed system, anaerobic and aerobic respiration would occur at the same rate. D) Because in a closed system, anaerobic respiration would generate more ATP than aerobic respiration.

A) Since nothing new can enter or leave the system, there is a negative relationship between glucose levels and ATP concentration.

Free energy decreases and the stability of a system increases during which type of reactions? A) Spontaneous B) Endergonic C) Never D) Nonspontaneous

A) Spontaneous

The end product(s) of photosynthesis in plants are oxygen and _______? A) Starch B) Chlorophyll C) Carbon dioxide D) Glucose E) Water

A) Starch

What is one function of triglycerols living organisms? A) Storage of fuel (energy) B) Synthesis of hormones C) Storing genetic information D) Catalysis of chemical reactions. E) Cell structure.

A) Storage of fuel (energy)

What are the two major roles of polysaccharides in living things? A) Storage of fuel (energy). B) Storage of water. C) Structure. D) Creating membranes. E) Storage of information.

A) Storage of fuel (energy). C) Structure Explain: Some polysaccharides store sugars to be broken down for energy, e.g. glycogen and starch. Others are structural molecules in plant cell walls (like cellulose).

In an eukaryotic cell, electron transport takes place within: A) The inner mitochondrial membrane B) The cytoplasm C) The mitochondrial matrix D) The plasma membrane E) The outer mitochondrial membrane

A) The inner mitochondrial membrane

Which of the following is false about the process of DNA replication? A) The leading strand is synthesized as a series of segments called the Okazaki fragments. B) Both leading and lagging strands are elongated in 5' to 3' direction. C) DNA polymerase can only add nucleotides to the free 3' end of a growing DNA strand or a primer. D) DNA ligase joins all the Okazaki fragments into a continuous DNA strand.

A) The leading strand is synthesized as a series of segments called the Okazaki fragments.

The genetic code is redundant. What is meant by this statement? A) The same amino acid can be made by more than one codon. B) A single codon can specify the addition of more than one amino acid. C) The genetic code is the same sequence when read forward and backward. D) Members of the same species have identical genomes. E) The genetic code is different for different domains of organisms.

A) The same amino acid can be made by more than one codon.

Imagine that there are 25 different species of protists living in a tide pool. Some of these species reproduce both sexually and asexually, and some of them can reproduce only asexually. The pool gradually becomes infested with disease-causing viruses and bacteria. Which species are more likely to survive and/or thrive in the changing environment? A) The sexually reproducing species. B) The asexually reproducing species. C) Sexually and asexually reproducing species are equally likely to survive and/or thrive. D) Neither species will be able to surive and/or thrive.

A) The sexually reproducing species.

Why is CO2 produced as a product in alcoholic fermentation, but not in lactic acid fermentation? A) There is not carbon remaining to form the byproduct CO2 in lactic acid fermentation. B) The lactate consumes the CO2 in lactic acid fermentation. C) The NADH consumes the CO2 in lactic acid fermentation. D) Glycolysis prevents CO2 formation in lactic acid fermentation.

A) There is not carbon remaining to form the byproduct CO2 in lactic acid fermentation. Explain: In alcoholic fermentation, pyruvate is converted to ethanol. The by product is CO2. In lactic acid fermentation, pyruvate is converted in to lactate. There is not carbon remaining to form the byproduct CO2 in this reaction. Different organisms, different methods for recycling the electron carrier (NAD).

Why can certain types of viruses promote cancer in humans especially? A) They insert their DNA into the host genome and can cause massive damage. B) The viruses cause silent mutations in the host's genome. C) The body overcompensates for damaged organs and mass produce cells. D) The virus can mutate an oncogene into a proto-oncogene. E) The medicine used to treat the virus is what causes most cancers.

A) They insert their DNA into the host genome and can cause massive damage.

What role do cofactors play, specifically coenzymes, in enzymatic reactions? A) They temporarily bind to an enzyme to change its shape or configuration to assist a reaction. B) They function as allosteric regulators via the process of cooperativity. C) They are usually carbohydrate-based enzymes that regulate reactions. D) They temporarily bind to a substrate to change its shape or configuration to assist a reaction. E) They function to change the free energy released from the reaction.

A) They temporarily bind to an enzyme to change its shape or configuration to assist a reaction.

DNA replication is a complex process involving many enzymes. Mutations affecting any one of these enzymes is likely to lead to a negative disruption in DNA replication. You are working with a mutated strain of E. coli that cannot accurately replicate its DNA. The major issue is that the DNA remains in a super coiled state, inhibiting the replication fork from moving forward. The mutation is most likely in a gene encoding which of the following components of DNA replication: A) Topoisomerase B) Helicase C) DNA pol III or I D) DNA ligase E) Primase

A) Topoisomerase

What was the most significant conclusion that Gregor Mendel drew from his experiments with pea plants? A) Traits are inherited in discrete units and are not the result of "blending." B) Genes are composed of DNA. C) White pea plants were less evolutionary fit than purple pea plants. D) There is considerable genetic variation in garden peas. E) Recessive genes occur more frequently in the F1 generation than do dominant ones.

A) Traits are inherited in discrete units and are not the result of "blending."

Translation is a molecular process that A) Uses the information in an mRNA sequence to synthesize a protein B) Uses the information in a mRNA sequence to synthesize a new DNA sequence C) Uses the information in a rRNA sequence to synthesize a protein D) Uses the information in a DNA sequence to synthesize an mRNA sequence E) Uses the information in a DNA sequence to synthesize a new DNA sequence

A) Uses the information in an mRNA sequence to synthesize a protein

Which of the following is true about isomers? A) Vision is partly based off of geometric isomers changing from cis to trans isomers (and back). B) Enantiomers can be superimposed upon one another C) All enantiomers of a molecule will perform the same biological function. D) Structural isomers have the same arrangement but different molecular formula. E) Cis-trans geometric isomers are mirror images of each other.

A) Vision is partly based off of geometric isomers changing from cis to trans isomers (and back).

You have been hired by the Texas State Wine and Spirits Council. Your job is to test different sugars at different concentrations to see which produces the best vintage of red wine. Your first job is to prepare 750 mL of 3.5 M fructose. How will you do this? **C = 12, H = 1, O = 16**

Add 472.5 grams to 0.75 L of water. Work: MW = 6(12) + 12(1) + 6(16) = 180 180 x 3.5 = 630 630 x 7.5 L = 472.5

Boric acid, shown above, can be used in a number of medical applications. Very dilute solutions can treat both bacterial and fungal infections. How many grams of boric acid would be required to make 4 L of a 0.5 M solution? (Atomic mass of boron = 11, oxygen = 16, hydrogen = 1) A) 434 g B) 143.2 g C) 43.4 g D) 62 g E) 20.5 g

B) 143.2 g Explanation: MW = 11(1) + 16(3) + 3 = 62 g/mol 62 x 7 = 437 437 x 0.33 = 143.2 g

Furmi, the "normal" tailed orange boy, and Adi, the Manx Tortie girl, are both fixed (neutered/spayed). However, if they weren't and had a litter of kittens, which would inevitably be adorable, how many would NOT have a tail? (Remember, the Manx trait is an example of a dominant lethal allele.) Also, no, there is no information missing from this question. A) 25% B) 50% C) 0% D) 75% E) 100%

B) 50%

Axolotls have 14 chromosome pairs (2n = 28), how many sister chromatids will be found in a nucleus at prophase of mitosis? A) 28 B) 56 C) 14 D) 112 E) 7

B) 56

Which of the following changes would cause a frameshift mutation? A) An insertion of 12 codons. B) A duplication of 14 nucleotides. C) A substitution of G instead of the "normal" T. D) A deletion of 12 nucleotides E) None of these would cause a frameshift mutation.

B) A duplication of 14 nucleotides.

The genetic code is essentially the same for all organisms. From this, one can logically assume which of the following statements to be true. A) Different organisms have different types of amino acids. B) A gene from one organism can be introduced in and expressed by another organism. C) The same codons in different organisms translate into different amino acids. D) The code cannot function with any mistakes. E) Each domain of life each use their own stop codon, which is why there are 3 different stop codons.

B) A gene from one organism can be introduced in and expressed by another organism.

Which of the following is NOT true about DNA? A) Nearly every single organism alive uses the same genetic material (A, C, G, Ts) B) All DNA mutations negatively affect the fitness of an organism. C) DNA is synthesized in the 5' to 3' direction. D) The amount of DNA present in a genome varies considerably among species. E) Breaking a G-C pair is harder than breaking a T-A pair.

B) All DNA mutations negatively affect the fitness of an organism.

Which of the following is not a tenet of the cell theory? A) Cells can only arise by division from a preexisting cell. B) All cells have a nucleus. C) All organisms are composed of one or more cells. D) The cell is structural unit of life.

B) All cells have a nucleus.

Which process(es) is/are not found in a photosynthetic cell? A) Oxidative phosphorylation B) All of the options occur in a photosynthetic cell. C) Glycolysis, the citric acid cycle, and oxidative phosphorylation D) The citric acid cycle E) Glycolysis

B) All of the options occur in a photosynthetic cell.

Which of the following was NOT addressed by Darwin in On the Origin of Species? A) All species produce more offspring than the environment can support. B) An account for the origin of life on earth. C) Descent with modification D) Heritable variation exists in most species. E) Differential reproductive success.

B) An account for the origin of life on earth.

Which of the following statements correctly describes the preferred tonicity conditions for typical plant and animal cells? A) An animal cell in an isotonic solution, a plant cell in a hypertonic solution. B) An animal cell in an isotonic solution, a plant cell in a hypotonic solution. C) An animal cell in a hypertonic solution, a plant cell in an isotonic solution. D) An animal cell in a hypotonic solution, a plant cell in an isotonic solution. E) An animal cell in a hypertonic solution, a plant cell in a hypotonic solution.

B) An animal cell in an isotonic solution, a plant cell in a hypotonic solution.

What is depicted in the pathway above? A) An example of competitive inhibition. B) An example of allosteric regulation via feedback inhibition. C) An example of enzyme denaturation. D) An example of allosteric regulation via cooperativity. E) An example of a "normal", positively regulated enzymatic reaction.

B) An example of allosteric regulation via feedback inhibition.

Which of the following comparisons between the prokaryotic domains is CORRECT? A) Archaea can be multicellular, bacteria are only unicellular. B) Archaea have no known species that are human pathogens, bacteria have several. C) Archaea have linear chromosomes, bacteria have circular ones. D) Archaea are only found in aquatic environments. E) Archaea have membrane-enclosed organelles but bacteria do not.

B) Archaea have no known species that are human pathogens, bacteria have several.

During the first stage in the light-independent reactions of photosynthesis, carbon dioxide is said to be "fixed". What does this mean? A) CO2 is converted to an inorganic compound, from an organic precursor. B) CO2 is converted to an organic compound, from an inorganic precursor. C) CO2 is released from the plant as a waste product. D) CO2 is converted to RuBisCO. E) CO2 is used first as an electron donor, then an electron acceptor.

B) CO2 is converted to an organic compound, from an inorganic precursor.

What is a predominant role of phospholipids in organisms? A) Synthesizing hormones B) Creating membranes C) Catalysis of chemical reactions D) Storage of fuel (energy)

B) Creating membranes Explain: Phospholipids are used to build membranes.

Which of the following is an example of applied science? A) Understanding the phylogenetic relationship between E. coli strands. B) Developing a successful vaccine for the novel Corona virus. C) Determining the ratio of purple to white flower colors in pea plants. D) Figuring out what atoms are composed of. E) Tracking down "patient zero" for the novel Corona virus.

B) Developing a successful vaccine for the novel Corona virus. Explanation: Applied science is technology, so a new vaccine would be the only technology option.

DNA replication is said to be semiconservative. What does this mean? A) One strand of the new double helix is made of DNA and the other strand is made of RNA. B) Each new double helix consists of one old and one new strand. C) One of the two resulting double helices is made of two old strands, and the other is made of two new strands. D) The old double helix is degraded and half of its nucleotides are used in the construction of two new double helices. E) Half of the old strand is degraded and half is used as a template for the replication of a new strand.

B) Each new double helix consists of one old and one new strand.

A primary function of DNA in somatic cells is A) Converting glucose to pyruvate B) Encoding mRNAs and subsequently proteins C) DNA has no function in somatic cells D) Synthesizing membrane phospholipids

B) Encoding mRNAs and subsequently proteins

Which of the following statements is correct regarding entropy at the molecular level? A) A clean room has more entropy than a messy room. B) Entropy increases when a substance is broken down into parts. C) Entropy of a gaseous state is less than the entropy of a liquid state. D) Entropy increases as temperature decreases. E) A sand castle has more entropy than "unassembled" sand lying on the beach.

B) Entropy increases when a substance is broken down into parts.

A chemical reaction that has a negative delta G is best described as ________ while one with a positive delta G is _______. A) Exocytic, endocytic B) Exergonic, endergonic C) Endergonic, exergonic D) Nonspontaneous, spontaneous

B) Exergonic, endergonic

The passage of water molecules across cell membranes through aquaporins is an example of: A) Simple diffusion B) Facilitated diffusion C) Active transport D) Endocytosis

B) Facilitated diffusion Explain: Aquaporins are channels selective for water molecules, so this is an example of facilitated diffusion.

Which of the summary statements below describe the results of the following reaction? C6H12O6 + 6 O2 --> 6 CO2 + 6 H2O + Energy A) Carbon dioxide is reduced and oxygen is oxidized. B) Glucose is oxidized and oxygen is reduced. C) Oxygen is reduced and carbon dioxide is oxidized. D) Oxygen is oxidized and H2O is reduced. E) Glucose is reduced and oxygen is oxidized.

B) Glucose is oxidized and oxygen is reduced.

Fermentation relies on which of the following stage(s) of cellular respiration? A) Electron transport chain B) Glycolysis C) Krebs' Cycle D) Glycolysis and Krebs' Cycle E) Glycolysis and Electron transport chain

B) Glycolysis Explain: Glycolysis occurs under anaerobic conditions (lack of oxygen), like fermentation. Krebs' cycle and electron transport chain occurs under aerobic conditions (requires oxygen).

If you place a cuvette of chlorophyll in a spectrophotometer, and shine different wavelengths of light on it, which wavelength of light should give you the highest transmittance reading? A) White B) Green C) Blue D) Red E) Purple

B) Green

The drug thalidomide has two different enantiomers, which of the following statements is NOT true about this drug? A) It was once used to cure morning sickness and restlessness for pregnant women. B) If given the appropriate enantiomer, pregnant women can now safely take it. C) Today it is a multi-hundred-million-dollar industry. D) Can be used to treat diseases including leprosy and breast cancer. E) One version is a teratogenic drug (related to birth defects)

B) If given the appropriate enantiomer, pregnant women can now safely take it. Explanation: It is still bad for pregnant women.

Both cellular respiration and photosynthesis rely on electron transport chains embedded in membranes to produce ATP molecules. Which of the following correctly describes the difference between the two types of electron transport chains? A) Both b. and c. B) In cellular respiration, the electron source originates in energy-rich food; in photosynthesis, the electron source is water. C) In cellular respiration, the electron source is water; in photosynthesis, the electron source is CO2. D) In cellular respiration, the final electron acceptor is CO2; in photosynthesis, the final electron acceptor is NADPH.

B) In cellular respiration, the electron source originates in energy-rich food; in photosynthesis, the electron source is water.

Celery stalks that are immersed in fresh water for several hours become stiff and hard. Similar stalks left in a salt solution become limp and soft. From this we can determine that the fresh water A) Is isotonic and the salt solution is hypertonic to the cells of the celery stalks. B) Is hypotonic and the salt solution is hypertonic to the cells of the celery stalks. C) Is hypertonic and the salt solution is hypotonic to the cells of the celery stalks. D) And the salt solution are both hypotonic to the cells of the celery stalks. E) And the salt solution are both hypertonic to the cells of the celery stalks.

B) Is hypotonic and the salt solution is hypertonic to the cells of the celery stalks.

Quaternary structure A) Is only possible for identical polypeptide subunits. B) Is the overall shape of two or more polypeptide chains that associate to form one functional macromolecule. C) Is a protein's overall size. D) Is observed for proteins such as a-keratin.

B) Is the overall shape of two or more polypeptide chains that associate to form one functional macromolecule.

In liver cells, the inner mitochondrial membranes are about five times the area of the outer mitochondrial membranes. What does this folding cause? A) It allows for an increased rate of glycolysis. B) It increases the surface area for oxidative phosphorylation. C) It allows for an increased rate of the citric acid cycle. D) It increases the surface area for substrate-level phosphorylation. E) It causes a decreased rate for glycolysis as more distance has to be covered.

B) It increases the surface area for oxidative phosphorylation.

Which of the following is true about the citric acid cycle? A) It requires pyruvate as an input. B) It requires oxygen. C) It occurs in the cytosol (cytoplasm) of prokaryotic organisms. D) It generates the most ATP among all respiration processes. E) It does not release/generate any CO2.

B) It requires oxygen C) It occurs in the cytosol (cytoplasm) of prokaryotic organisms.

Which of the following does NOT describe a biological membrane? A) Carbohydrate portion of its glycoproteins are positioned on the extracellular face. B) Its protein and lipid components are symmetrically distributed. C) It is a collage of proteins, embedded in the fluid matrix of the lipid bilayer. D) Its components get transported through vesicles, whose outside face becomes continuous with the membrane cytoplasmic face.

B) Its protein and lipid components are symmetrically distributed.

People (not just scientists) have concluded that, "The sun will rise tomorrow." This conclusion is based on inductive reasoning. This means that... A) This conclusion cannot be rejected B) Many observations have led us to infer this conclusion about the sun. C) This conclusion was the result of an accidental discovery. D) This result is from many controlled experiments and was proven to be true. E) This hypothesis has been rigorously scientifically tested.

B) Many observations have led us to infer this conclusion about the sun.

Which of the following is NOT considered part of the cell's endomembrane system? A) Nuclear envelope B) Mitochondria C) Endoplasmic reticulum (ER) D) Vesicles E) Lysosomes

B) Mitochondria

The microbiologist Frederick Griffith studied Streptococcus pneumonia and used mice to test different strains of the bacterium. What conclusion did his work lead to? A) Infecting mice with nonpathogenic strains of bacteria makes them resistant to pathogenic strains. B) Mixing a heat-killed pathogenic strain of bacteria with a living nonpathogenic strain can convert some of the living cells into the pathogenic form. C) Mice infected with a pathogenic strain of bacteria can spread the infection to other mice. D) Mixing a heat-killed nonpathogenic strain of bacteria with a living pathogenic strain makes the pathogenic strain nonpathogenic. E) No matter the strain of bacteria injected the mice died from pneumonia.

B) Mixing a heat-killed pathogenic strain of bacteria with a living nonpathogenic strain can convert some of the living cells into the pathogenic form.

Electronegativity is the attraction of a particular atom for electrons in a covalent bond. Which of the following is TRUE about the general trend of electronegativity that we discussed in class? A) The further the electrons are from the nucleus, the more electronegative it is. B) More protons = more electronegativity C) More electrons = less electronegativity D) Less electrons = more electronegativity E) Less protons = more electronegativity

B) More protons = more electronegativity

We discussed a phenomenon called the Warburg Effect and its implications for glucose metabolism. Which of the following statements is false? A) Aerobic respiration produces much more ATP than anaerobic respiration. B) Oxidative phosphorylation is a much faster process than anaerobic respiration. C) Some cells, including cancer cells, prefer this process. D) Some cells, even in the presence of oxygen, will still undergo anaerobic respiration. E) Anaerobic respiration is a much faster process than oxidative phosphorylation.

B) Oxidative phosphorylation is a much faster process than anaerobic respiration.

ATP drives endergonic reactions by the process of __________, which is the __________. A) Hydrolysis, the breaking of a chemical bond by the addition of a water molecule. B) Phosphorylation, transferring a phosphate to other molecules. C) Dephosphorylation, the removing a phosphate group from an organic compound. D) Dehydration synthesis, the joining of two molecules by the removal of a water.

B) Phosphorylation, transferring a phosphate to other molecules.

The operation of the sodium potassium "pump" moves: A) Sodium ions into the cell and potassium ions out of the cell. B) Sodium ions out of the cell and potassium ions into the cell. C) Sodium and potassium ions into the mitochondria. D) Sodium and potassium ions out of the cell. E) Sodium and potassium ions into the cell.

B) Sodium ions out of the cell and potassium ions into the cell.

At Starbucks you take 3 packets of sugar and stir them into your cup of coffee until it disappears completely. In this example, the sugared coffee is the _________, and the sugar is the ________. A) Solvent; solute B) Solution; solute C) Solute; solution D) Acid; base E) Reactant; product

B) Solution; solute

Which of the following is FALSE about the relationship between catabolism and anabolism? A) Both pathways involve enzymatic activities. B) The ATP generated from anabolic reactions drives catabolic reactions. C) Products of catabolic processes provide substrates for the anabolic processes and vice versa. D) Catabolism is exergonic, while anabolism is endergonic.

B) The ATP generated from anabolic reactions drives catabolic reactions.

Which most accurately describes the why leaves change color in fall? A) Degraded chlorophyll changes into many other colors. B) The accessory pigments that were there the whole time become more visible as chlorophyll production slow/stops. C) Sugars from sap fill the leaves prior to winter. D) The production of more accessory pigments because of the cooler temps. E) The reduction in production of accessory pigments because of the cooler temps.

B) The accessory pigments that were there the whole time become more visible as chlorophyll production slow/stops.

Which of the following is FALSE regarding the DNA double helix? A) The two strands are stabilized by hydrogen bonds and base stacking interactions. B) The two strands are parallel. C) The two strands exhibit complementary base pairing. D) The four nitrogenous bases found in it are A, T, C, and G.

B) The two strands are parallel.

Which of the following is NOT true about hydrocarbons? A) They are absent (without other bonds) in most living organisms. B) They easily dissolve in water. C) They are only composed of hydrogens and carbons (but can make bonds with other compounds). D) They have hydrophobic regions.

B) They easily dissolve in water

For Mendel's pea plants, tall is completely dominant to dwarf. Two plants are crossed, one tall & one dwarf. They produce 47 tall offspring and 52 dwarf offspring. The genotypes of the parents in this cross are ______ (tall parent) and __________ (dwarf parent). A) TT, tt B) Tt, tt C) TT, Tt D) tt, TT E) Tt, TT

B) Tt, tt

Which of the following mutations would usually be the least problematic? A) a single base-pair indel B) a single codon indel C) a substitution in the 1st or 2nd position of a codon D) an addition of five nucleotides E) the addition of UGA the middle of the sequence

B) a single codon indel

An atom with 8 electrons in its outer shells tend to A) bond to itself. B) be chemically inert. C) be an anion in aqueous solution. D) bond to hydrogen in aqueous solution. E) be a cation in aqueous solution.

B) be chemically inert.

The chlorine in table salt has 17 protons, 18 neutrons, and 18 electrons and it carries a negative charge. This makes it a(n).... A) atom B) ion C) compound D) salt E) radioactive isotope

B) ion Explanation: An ion is an atom (or molecule) with more electrons than protons or fewer electrons than protons and carries an electrical charge. Table salt is a compound and a salt, but the chlorine inside it is not. While the chlorine is an atom, it would still be an atom if it did not have a charge and was not an ion. Electrons do not determine isotopes or radioactivity.

Crossing over of chromosomes normally takes place during which of the following processes? A) binary fission B) meiosis I C) mitosis D) mitosis and meiosis II E) meiosis II

B) meiosis I

The purpose of a spliceosome is to: A) add a poly A to mRNA B) remove introns from the premature mRNA C) add a 3' cap to mRNA D) remove mRNA from the nucleus E) link Okazaki fragments together on the lagging strand

B) remove introns from the premature mRNA

You are a forensic expert called into testify and validate the results of PCR from DNA found at a crime scene. You were asked to calculate how much DNA is present in the sample given that it was originally 600 base pairs long and 25 rounds of PCR were completed using a thermocycler. What is your answer? A) ~20 MB (~20 million basepairs) B) ~20 GB (~20 billion basepairs) C) ~15 KB (~15 thousand basepairs) D) ~20 KB (~20 thousand basepairs) E) ~33.5 MB (~33.5 million basepairs)

B) ~20 GB (~20 billion basepairs)

An unscrupulous team of fruit fly geneticists from LSU decide to cross two species of flies, one from Aggieland and another from that other school in Texas. The fruit fly population from A&M are homozygous dominant for both phenotypes having maroon eyes and the gig 'em appendange. The population from Austin are homozygous recessive with orange eyes and the hook 'em appendage. The cross results in the following offspring: 1934 Maroon eyes, Gig 'em appendage 120 Maroon eye, Hook 'em appendage 130 Orange eyes, Gig 'em appendage 1916 Orange eyes, Hook 'em appendage What is the recombination frequency in this unlikely dihybrid population of flies? A) ~3% B) ~6% C) ~94% D) ~47% E) 0%

B) ~6%

Terminals

Branches that are attached to only a single node...these are often used to represent taxa, or a single character vector of a matrix

How are sister chromatids held together?

By centromeres

Some radioactive iodide salts of sodium, including sodium iodide (NaI), have radiopharmaceutical uses, such as in the treatment of thyroid cancer and hyperthyroidism. You will treat your patient with a solution of NaI that you made by dissolving 12.3 g NaI in 200 mL of water. What is the molarity of the NaI solution that you will give your patient? A) 0.2 M B) 0.08 M C) 0.41 M D) 41 M E) 1 M

C) 0.41 M Explanation: MW = 149.9 g/mol 12.3/149.9 = 0.082 x 5 = 0.41 M

If the H+ ion concentration of a household item is 10^-3, how many OH- ions does it have? A) 10^-14 B) 10^-3 C) 10^-11 D) 10^-13

C) 10^-11

What is the molecular weight of glucose? C6H12O6 (C = 12; H = 1; O = 16) A) 432 g/mol B) 342 g/mol C) 180 g/mol D) 29 g/mol

C) 180 g/mol Explanation: MW = 6(12) + 12(1) + 6(16) = 180

Cytosine makes up 24% of the nucleotides in a sample of DNA from an organism. Approximately what percentage of the nucleotides in this sample will be guanine? A) 48% B) 52% C) 24% D) 12% E) 26%

C) 24%

An aqueous 2 M solution of hydrochloric acid is prepared with a total volume of 0.350 L (350 ml). The molecular weight of hydrochloric acid is 36.46 g/mol. How much hydrochloric acid (in grams) is needed for the solution? A) 34.5 g B) 12.8 g C) 25.5 g D) 72.9 g

C) 25.5 g Explanation: 1) MW = 36.46 g 2) We need 2 M so 36.46 x 2 = 72.92 g/L 3) We only need 0.35 L or 350 mL so 75.92 x 0.35 = 25.5g

In a particular piece of DNA, 72% of the nucleotides have paired up with their complement nucleotide via a triple hydrogen bond. According to Chargaff's Rule, how many guanines will this piece of DNA have in it? A) 14% B) 28% C) 36% D) 24% E) 18%

C) 36%

A 0.250 L (250 mL) sample of seawater contains 77.4 grams of NaCl, which has a molecular weight of 58.44 g/mol. What is the molar concentration of the NaCl in the seawater? A) 0.631 M B) 310 M C) 5.30 M D) 1.0 M

C) 5.30 M Explanation: 1) MW = 58.44 grams for 1 M 2) We have 250 mL or ) 0.25 L of NaCl solution. 3) IN that 250 mL, we have 77.4 g of NaCl so we are already WAY over 1 M, right? How much so.... 4) Well, 77.4/58.44 = 1.32 M, but that's if we had a full liter, but we only have 1/4 liter (250 mL) 5) 1.32 x 4 = 5.3 M

What molecules are made during light reactions of photosynthesis that are required for use during the light independent reactions to make carbohydrates? A) Glucose and oxygen B) ADP and NADP+ C) ATP and NADPH D) Water and oxygen E) Sugar and oxygen

C) ATP and NADPH

Element imbalances can cause significant issues for all living organisms. What does a deficiency of iron (Fe) cause in humans? Pick the condition that was specifically addressed in class? A) Hyperthyroidism B) A rare type of blood cancer C) Anemia D) Goiters E) Birth defects

C) Anemia Explanation: Low levels of iron: anemia High levels of iron: blood cancer

Which of the following allows the fast and efficient bulk transport of water across cell membranes? A) ATP B) The sodium-potassium pump C) Aquaporins D) Cholesterol

C) Aquaporins

The pH of an item is 13. Is this acidic or basic and how many H+ ions does it have? A) Basic 10^-1 B) Acidic 10^-1 C) Basic 10^-13 D) Acidic 10^-13

C) Basic 10^-13

A mutation occurs that affects the process of DNA replication. After the DNA is replicated, each new molecule consists of one "normal" strand paired with a strand composed of several discontinuous fragments of DNA that are a few hundred base pairs long. The mutation is most likely in a gene encoding which of the following components of DNA replication: A) DNA pol III OR i B) Topoisomerase C) DNA ligase D) Primase E) Helicase

C) DNA ligase

What do we call a reaction where water is removed? A) Futile reaction B) Catalyzed reaction C) Dehydration reaction D) Peptide reaction E) Hydrolysis reaction

C) Dehydration reaction

The construction of macromolecules requires a series of ___________ reactions that form ___________ bonds between monomers. A) Cohesive; ionic B) Denaturation; ionic C) Dehydration; covalent D) Hydrophilic; covalent E) Hydrolysis; polymer

C) Dehydration; covalent

Different types of cells in your body look and function differently. The basis for these differences is A) Different types of DNA in each cell B) Different amounts of DNA in each cell C) Different patterns of gene expression D) Different number of chromosomes

C) Different patterns of gene expression

Which of the following shows the BEST example for the indicated energy transformation? A) Climbing and finally reaching the mountain top: potential energy to kinetic energy. B) Food digestion to regulate body temperature: kinetic energy --> chemical energy C) Dropping a pencil --> potential energy --> kinetic energy D) Burning charcoal to boil water: thermal energy --> chemical energy

C) Dropping a pencil --> potential energy --> kinetic energy

Select the CORRECT match of transport across a plasma membrane and description. A) Facilitated diffusion -- movement against a concentration gradient (low to high) through a specific transport protein (no energy required). B) Osmosis -- movement of water from areas of high solute concentration to areas of low solute concentration. C) Exocytosis -- transport vesicles move to the plasma membrane, fuse with it, and release their contents out of the cell. D) Active transport -- movement of a large quantity of water molecules across the hydrophobic plasma membrane. E) Pinocytosis -- ingestion of small, solid particles into the cell.

C) Exocytosis -- transport vesicles move to the plasma membrane, fuse with it, and release their contents out of the cell.

Of the following cell components, which group comprises the endomembrane system? A) Peroxisomes, mitochondria, endoplasmic reticulum B) Lysosome, vacuole, peroxisomes C) Golgi apparatus, vacuole, endoplasmic reticulum D) Nuclear membrane, nucleolus, Golgi apparatus

C) Golgi apparatus, vacuole, endoplasmic reticulum

If you were to observe the activity of methylated DNA, you would expect it to A) Be replicating nearly continuously. B) Be very actively transcribed and translated. C) Have turned off or slowed down the process of transcription. D) Induce protein synthesis by not allowing repressors to bind to it. E) Be unwinding in preparation for protein synthesis.

C) Have turned off or slowed down the process of transcription.

Which of the following statements is true? A) Hydrogen bonds between amino acid side chains (R groups) are responsible for stabilizing a protein's secondary structure. B) A B-pleated sheet is a structure formed by hydrogen bonds between every fourth amino acid in a polypeptide chain. C) Hydrogen bonds are generally weak, yet they stabilize protein secondary structures because they are found throughout the polypeptide backbone. D) A-helices make up most fibrous proteins such as the silk protein of a spider's web.

C) Hydrogen bonds are generally weak, yet they stabilize protein secondary structures because they are found throughout the polypeptide backbone.

What functional group that you learned about are there a lot of in sugars? A) Carboxyl groups B) Phosphate groups C) Hydroxyl groups D) Methyl groups E) Amino group

C) Hydroxyl group Explanation: Sugars always contain multiple hydroxyl groups. They also have 1 (or, if they are disaccharides, 2) carbonyls.

Which of the following characteristics is NOT shared by bacteria and archaea? A) Presence of ribosomes B) Presence of plasma membrane C) Identical cell wall composition D) Absence of nuclear membrane

C) Identical cell wall composition

When a macromolecule goes through complete hydrolysis, the products are A) Salts. B) Individual atoms. C) Monomers D) Ions. E) Polymers

C) Monomers

One way cells control the potential "explosions" of respiration is by separating in time and space, the removal of electrons from food and their reaction with O2. In what molecule(s) are the electrons temporarily held? A) ATP B) CO2 C) NADH and FADH2 D) H+ E) Proteins

C) NADH and FADH2

In pyruvate oxidation, all of the following occur EXCEPT which of these: A) A carboxyl group is removed from a molecule of pyruvate B) Pyruvate is converted to a two-carbon molecule attached to a CoA. C) NADH is oxidized to NAD+ D) Coenzyme A attaches to the newly formed acetyl group resulting in acetyl CoA. E) Carbon dioxide is released.

C) NADH is oxidized to NAD+ Explain: Instead, NAD+ is reduced to NADH.

The Hershey and Chase experiments using the T2 bacteriophage and E.coli provided very strong evidence that: A) Genetic traits can be transformed between differential bacterial strains. B) DNA replication is semi-conservative. C) Nucleic acids are the hereditary material used by viruses. D) Proteins are the hereditary material used by viruses. E) DNA is composed of four different nitrogenous bases.

C) Nucleic acids are the hereditary material used by viruses.

Which of the following is NOT a molecule? A) NaCl B) H2O C) O D) None of these E) C6H12O6 F) O2

C) O

Which of the following is NOT a compound? A) NaCl B) H2O C) O2 D) HCl E) None of these F) C6H12O6

C) O2

Mitochondria are found A) Only in animal cells B) Only in prokaryotes C) Only in eukaryotes D) In all cells

C) Only in eukaryotes

The majority of the membrane is made up of: A) Cholesterol B) Proteins C) Phospholipids D) Carbohydrates

C) Phospholipids Explain: The phospholipids make up most of the membrane. Protein and carbohydrates account for a small percentage.

Nearly all enzymes are what kind of biomolecule? A) Nucleic acids B) Lipids C) Proteins D) Polymers E) Carbs

C) Proteins

Which of the following is TRUE about RNA as compared to DNA? A) RNA is made up of five different nucleotides. B) RNA includes both coding and non-coding base pairs. C) RNA is exceptionally unstable. D) RNA is always double-stranded. E) RNA uses the nucleotide thymine in place of uracil.

C) RNA is exceptionally unstable.

What is the main enzyme used during transcription? A) Helicase B) DNA polymerase C) RNA polymerase D) Ligase

C) RNA polymerase

Covid-19 is a novel strain belonging to a large group of coronaviruses. Their effective route of entry into host cells is attributed to "spike" proteins that have a very unique morphology to them. These proteins on the surface of the virus bind to the host cell's plasma membrane and admit the virus. This type of cell entry mechanisms is called: A) Active transport B) Pinocytosis C) Receptor-mediated endocytosis D) Facilitated transport (or facilitated diffusion) E) Receptor-mediated exocytosis

C) Receptor-mediated endocytosis

Which of the following does not occur during mitosis? A) Separation of sister chromatids B) Spindle formation C) Replication of the DNA D) Nuclear envelope disappears and reappears E) Condensation of the chromosomes

C) Replication of the DNA

Despite the human genome being significantly larger than the genome of bacteria such as E.coli, it does not take a human cell significantly longer to replicate its genome. Why? A) The antiparallel orientation of the DNA in humans speeds it up. B) Human chromosomes are linear while the E.coli chromosome is circular. C) Replication on human chromosomes use multiple origins of replication and move in both directions along the DNA. D) Replication is more evolved in human cells as compared to E.coli cells. E) The enzymes in human replication are faster.

C) Replication on human chromosomes use multiple origins of replication and move in both directions along the DNA.

Most of the enzymes in our bodies function at or very close to neutral (pH ~ 7.0). There are a few exceptions. Which area of your body should be exception to this "rule"? A) Skin B) Heart/Blood C) Stomach D) Brain

C) Stomach

Pick the metabolic process that is endergonic. A) Catabolism of glucose B) Transition of water into ice. C) Synthesis of ATP and ADP. D) Break down of polypeptide into amino acids. E) Hydrolysis of ATP to ADP.

C) Synthesis of ATP to ADP.

A null hypothesis states that: A) There is no control group in this type of experiment B) The dependent variable has no effect on the independent variable. C) The independent variable has no effect on the dependent variable. D) There is no hypothesis that can be made.

C) The independent variable has no effect on the dependent variable.

How will a healthy individual's ATP production change during an eight-hour fast? A) The individual's ATP production will decrease significantly. B) The individual's ATP production will increase significantly. C) The individual's ATP production will not change significantly.

C) The individual's ATP production will not change significantly.

When you put aspirated spinach leaf disks in a beaker of sodium bicarbonate and place them in different colors of light, some disks float faster & more abundantly than others dependent on what color, if any, they were in. What made them able to float? A) The green light condition/treatment B) The release of O2 from the process of respiration. C) The release of O2 from the process of photosynthesis. D) The release of CO2 from the process of photosynthesis.

C) The release of O2 from the process of photosynthesis.

The genetic code is essentially the same for all organisms. From this, one can logically assume which of the following statements to be most reasonable? A) The code cannot function with any mutations. B) Different organisms have different types of amino acids. C) The same codons in different organisms translate into the same amino acids. D) A gene from one organism that was introduced into a different organism would never function properly. E) Each domain of life each uses their own stop codon; three domains of life = three stop codons.

C) The same codons in different organisms translate into the same amino acids.

Why can viruses cause mutations in their host's genome? A) The body overcompensates for damaged organs and mass produces cells (meaning more and faster DNA replication). B) The viruses insert their proteins into the host's genome and can cause significant muations, sometimes even leading to cancer. C) The virues insert their DNA/RNA into the host's genome and can cause significant mutations, sometimes even leading to cancer. D) The virus can cause abnormal T-T bonding (thymine dimers). E) The medicine used to treat the virus is what causes most issues/problems in the host.

C) The virues insert their DNA/RNA into the host's genome and can cause significant mutations, sometimes even leading to cancer.

Cellulose and chitin are both important for the following reason: A) They are both used as massive energy reserves for cells. B) They are vital signaling molecules (hormones). C) They provide structural support for plants and fungi, respectively. D) They are both major parts of plasma membrane structure. E) They both aid in transcription (DNA --> RNA).

C) They provide structural support for plants and fungi, respectively.

If you discovered a new organism and noticed that it uses anaerobic respiration but does NOT produce CO2 as a byproduct, what assumption could you make? A) This organism is most likely a prokaryote. B) This organism is most likely generating the majority of its ATP via chemiosmosis. C) This organism is most likely producing lactic acid. D) This organism is most likely producing ethanol. E) This organism is most likely an eukaryote.

C) This organism is most likely producing lactic acid.

What is the main function of topoisomerase? A) To degrade damaged DNA molecules B) To add nucleotides to the 3' end of a growing DNA strand C) To nick the DNA to relieve coiling before/during replication D) To rejoin the two DNA strands after replication E) To seal together the Okazaki fragments found in the lagging strand

C) To nick the DNA to relieve coiling before/during replication

The process of converting DNA to RNA is called ______. A) Translation B) Replication C) Transcription

C) Transcription

During photosystem II (PSII), oxygen is released as a waste product. What is the source of this oxygen? A) CO2 B) The atmosphere C) Water D) Simple sugars E) PSI

C) Water

Pick the metabolic process that is exergonic. A) going from amino acids to a polypeptide B) going from a polypeptide to a protein C) going from a polypeptide to amino acids D) going from glucose to chitin E) going from CO2 to glucose

C) going from a polypeptide to amino acids

If you were to observe the activity of methylated DNA, you would expect it to A) be replicating nearly continuously. B) be very actively transcribed and translated. C) have turned off or slowed down the process of transcription. D) induce protein synthesis by not allowing repressors to bind to it. E) be unwinding in preparation for protein synthesis.

C) have turned off or slowed down the process of transcription.

A sexually reproducing animal has two unlinked genes, one for head shape (H) and one for tail length (T). Its genotype is Hhtt. With "normal" meiosis, which of the following genotypes is possible in a gamete from this organism? A) tt B) t C) ht D) Hh E) Hhtt

C) ht

In its natural, freshwater environment, Paramecium constantly gains water from its environmental and must use a special organelle called a contractile vacuole to pump water from its cytoplasm. This gain of water occurs because the surrounding freshwater is _________ when compared to the cytoplasm of the Paramecium. A) hypertonic B) equitonic C) hypotonic D) isotonic E) supersonic

C) hypotonic

Compared to heterochromatin, euchromatin is A) rich in actively expressed genes and comprised of densely packed nucleosomes. B) poor in actively expressed genes and comprised of densely packed nucleosomes. C) rich in actively expressed genes and comprised of loosely packed nucleosomes. D) poor in actively expressed genes and comprised of loosely packed nucleosomes. E) rich in poorly expressed genes and completely lacking nucleosomes

C) rich in actively expressed genes and comprised of loosely packed nucleosomes.

One way to ease a sore throat is to mix warm water and salt together and use it to gargle (wash it around the back of your throat but not swallow). In this example, the salt-water is the __________ and the water (pre-salted) is the ________. A) reactant; product B) solvent; solution C) solution; solvent D) solute; solvent E) acid; base

C) solution; solvent Explanation: A solution is a mixture of a solute and a solvent. The water (pre-salted) is the solvent. A solvent is a liquid which dissolved a solute. The salt is a solute, which is a substance that is dissolved.

Saline is a very common solution used for intravenous injection made with approximately 0.16 M solution of sodium chloride (MW NaCl = 58.44 g/mol) in water. How much sodium chloride is needed to prepare 500 mL of saline solution? A) 46.8 grams B) 9.4 grams C) 4.68 grams D) 2.3 grams E) 58.4 grams

C. 4.68 grams Explanation: MW = 58.44 g/mol and we need 16% of that. 0.16(58.44) = 9.35 but we only knew half of that. 9.35/2 = 4.68 grams

Interphase

Cell grows, performs its normal functions, and prepares for division; consists of G1, S, and G2 phases

All living organisms have ________.

Cells (single or many)

Albinism is inherited as an autosomal recessive trait. A man and woman both show normal pigmentation, but both have one parent who has albinism (without melanin pigmentation). What is the probability that their first child will have albinism? A) 0 B) 3/4 C) 1 D) 1/4 E) 1/2

D) 1/4

Flower color in snapdragons is an example of incomplete dominance. When a pure bred purple-flowered plant is crossed with a pure bred white-flowered plant, the entire F1 generation has pink flowers. If a F1 pink-flowered plant is crossed with another F1 pink-flowered plant, the phenotypic ratio in the F2 generation will be __________. A) 25% white and 75% purple B) 100% purple C) 50% pink and 50% purple D) 25% white, 50% pink, 25% purple E) 100% pink

D) 25% white, 50% pink, 25% purple

Phosphorous is an important element for biological organisms. What is its valency (or combining powers) as we defined in biology class? A) 5 B) 15 C) 0 D) 3 E) 8

D) 3 Explanation: How many bonds does it want to make? In this case, phosphorous will create 3 bonds.

Based on your understanding of biological macromolecules, the number of unique monomer molecules that is common to living organisms is closest to which number? A) 5,000 B) 50 C) >500,000 D) 500 E) 50,000

D) 500

Oxidative phosphorylation accounts for approximately what percentage of the ATP formed by the entire processes of glucose metabolism (cellular respiration)? A) 0% B) 10% C) 50% D) 90% E) 100%

D) 90%

Why is the one gene-one enzyme hypothesis is not entirely accurate? A) Some genes encode a piece of a protein, not the entire protein. B) Some genes encode functional RNA molecules, not polypeptides. C) Actually, it is entirely accurate. No modifications are needed. D) A and B only E) A, B, and C

D) A and B only

The primary barrier of biological membranes consists of A) A lipid monolayer B) Polysaccharides C) Fatty acids D) A lipid bilayer

D) A lipid bilayer

Charles Darwin was the first person to propose which of the following? A) That cells contain the hereditary information in the form of DNA. B) That evolution occurs C) That Earth is MUCH older than a few thousand years. D) A mechanism for evolution that was supported by evidence.

D) A mechanism for evolution that was supported by evidence.

A biologist is studying a cell that has enzymes, linear chromosomes, a plasma membrane, ribosomes, and mitochondria. Given only this information, this cell could be A) A bacteria cell but not a plant cell B) A bacteria cell or a plant cell C) An animal cell but not a plant cell D) A plant cell or an animal cell but not a bacteria cell. E) A plant cell but neither a bacteria cell nor an animal cell.

D) A plant cell or an animal cell but not a bacteria cell.

The most common usable form of energy in a cell is A) Phosphate B) Glucose C) deoxy-ATP D) ATP

D) ATP

On the basis of energy requirement, which does NOT belong in the group? A) Facilitated diffusion B) Passive transport C) Osmosis D) Active transport

D) Active transport

Most of the organic material in plants come from: A) Soil/dirt B) Water C) Sunlight D) Air

D) Air

There is a tremendous amount of variation among all life. However, small things unite everything from the smallest bacteria to the largest mammal. Which of the following is NOT one of those characteristics? A) All living organisms share and use the same 20 amino acids. B) All living organisms have and use DNA as their genetic code. C) All living organisms have ribosomes. D) All living organisms reproduce sexually.

D) All living organisms reproduce sexually. Explanation: This is not true because many organisms reproduce asexually.

Which of these could lead to conversion of a proto-oncogene into an oncogene? A) Translocation of proto-oncogenes near an active promoter. B) Amplification of a proto-oncogene through gene duplication. C) A point mutation in proto-oncogene or its regulatory elements. D) All of the above.

D) All of the above.

Which of the following is an example of post-transcriptional control of gene expression? A) Alternative splicing of exons B) mRNA degradation C) Protein processing and degradation D) All of the options.

D) All of the options.

Proteins are polymers of what type of molecule? A) Nucleic acids B) Sugars C) Carbohydrates D) Amino acids

D) Amino acids

Which of these molecules, if radioactively labeled, will be incorporated into a newly made protein? A) Nucleotides B) Monosaccharides C) Isoprene D) Amino acids E) Glycerol and fatty acids

D) Amino acids

Which of the following explains why humans can digest starch but not cellulose? A) Because humans lack the enzyme which recognizes a-1, 4-glycosidic linkages B) Because cellulose has a lot of branched regions making it difficult for the enzymes to find the cleavage points for complete digestion in humans C) Because starch has a lot of branched regions allowing a greater surface area for easy digestion in humans D) Because humans lack the enzyme which recognizes B-1, 4-glycosidic linkages

D) Because humans lack the enzyme which recognizes B-1, 4-glycosidic linkages

Biological fluids regulate pH values using _________. This is achieved by accepting _________ when pH is low and donating __________ when pH is high. A) Acids, OH- ions, H+ ions B) None of the above C) Buffers, OH- ions, H+ ions D) Buffers, H+ ions, H+ ions E) Bases, H+ ions, OH- ions

D) Buffers, H+ ions, H+ ions

What would be the molecular formula for a polymer made from eight glucose (C6H12O6) molecules linked together by dehydration reactions? A) C48H80O40 B) C48H96O48 C) C48H110O62 D) C48H82O41 E) None of the above

D) C48H82O41

Which of the following is NOT one of the four major elements that make up biological macromolecules? A) Carbon B) Hydrogen C) Nitrogen D) Calcium E) Oxygen

D) Calcium Explain: Living organisms are made of organic molecules, which have C, H, O, and N as the four major elements. Calcium is an important signaling molecule in biological systems, but is not itself a major component of biological macromolecules.

C = O Which type of functional group is depicted above, and which type of molecule would you find it in? A) Amino group, Amino acids B) Methyl group, proteins C) Hydroxyl group, carbohydrates D) Carbonyl group, carbohydrates E) Carboxyl group, fatty acids

D) Carbonyl group, carbohydrates Explanation: C = O is a carbonyl group, and they are found in sugars (aka carbohydrates)

What do animal cells have to the exclusion of plant cells? A) Chloroplasts B) Mitochondria C) Ribosomes D) Centrioles

D) Centrioles

What are the major absorbing pigments in photosynthesis? A) PSII B) Carotenoids C) Absorption spectra D) Chlorophylls

D) Chlorophylls

Hypothesis based science requires: A) Ignoring experimental evidence that does not fit with the original hypothesis B) Being able to test and prove your hypothesis C) That the hypothesis be correct D) Control groups

D) Control groups

Hydrolysis is involved in each of the following EXCEPT A) Conversion of glycogen to glucose B) Conversion of DNA to a mixture of four deoxynucleotides C) Conversion of triacylglycerol to a mixture of fatty acids and glycerol D) Conversion of glucose to starch

D) Conversion of glucose to starch

The PCR (polymerase chain reaction) protocol that is currently used in many laboratories was facilitated by the discovery of a bacterium called Thermus aquaticus in a hot spring inside Yellowstone National Park, in Wyoming. This organism contains a heat-stable form of DNA polymerase known as Taq polymerase, which continues to function even after it has been heated to 95ºC. Why would such a heat-stable polymerase be beneficial in PCR? A) Each cycle includes a "hot" saturation phase (95°C), which allows the primers to anneal to the target DNA. B) Each cycle includes a "hot" denaturation phase (95°C), which serves to sterilize the culture. C) Each cycle includes a "hot" denaturation phase (95°C), which activates the Taq polymerase. D) Each cycle includes a "hot" denaturation phase (95°C), which separates the hydrogen bonds that hold the strands of the template DNA together. E) It's not, the fluorescent raccoon was wrong.

D) Each cycle includes a "hot" denaturation phase (95°C), which separates the hydrogen bonds that hold the strands of the template DNA together.

If LaMarck was correct in his idea about the inheritance of acquired traits, which of the following would you be able to pass along to your offspring? A) Your missing big toe after you lost it in an unfortunate corn hole accident. B) A tattoo you got on your 30th birthday. C) Your natural blue eye color. D) Either the tattoo or the missing toe.

D) Either the tattoo or the missing toe. Explanation: The natural blue eye color is a heritable trait, which does not fall under LaMarck's theory.

Which of the following is true about enzymes? A) Without enzymes, reactions would never take place. B) Enzyme function is increased if the 3-D structure or conformation of an enzyme is altered. C) Increasing temperature will increase the activity of an enzyme, while decreasing temperature will decrease the activity of an enzyme. D) Enzymes increase the rate of chemical reaction by lowering the activation energy. E) Enzymes increase the rate of chemical by providing activation energy to the substrate.

D) Enzymes increase the rate of chemical reaction by lowering the activation energy.

Select the correct match of transportation across a plasma membrane and description. A) Active transport -- moves substances from the hypertonic side to the hypotonic side of a cell (no energy required) B) Phagocytosis -- expels undigested food particles out of the cell C) Osmosis -- movement of any type of substance along its concentration gradient D) Facilitated diffusion -- movement down concentration gradient through a specific transport protein (no energy required) E) Exocytosis -- creates vesicles to transport bulk amounts of liquids into the cell

D) Facilitated diffusion -- movement down concentration gradient through a specific transport protein (no energy required)

Which of the following statement regarding fat molecules is FALSE? A) Fats are constructed from a single glycerol molecule and, usually, three fatty acids. B) Fats have tremendous variation in use (not just energy storage). C) Fats vary in length, number, and location of double carbon bonds. D) Fats are hydrophilic. E) Fats that are liquid at room temperature are usually healthier choices.

D) Fats are hydrophilic.

Which of the following shows carbohydrates arranged according to increasing number of sugar units? A) Glucose, glycogen, maltose B) Ribose, galactose, starch C) Lactose, cellulose, galactose D) Fructose, sucrose, chitin

D) Fructose, sucrose, chitin

Both alpha- and beta-hemoglobin chains have mostly alpha-helices, and no beta-sheets. What forces or bonds are responsible for formation and stabilization of alpha-helices? A) Van der Waals interactions B) Covalent disulfide bonds C) Hydrophobic interactions D) Hydrogen bonds E) Ionic bonds

D) Hydrogen bonds

If prokaryotes do not have mitochondria, then where are the enzymes for cellular respiration located? A) Nuclear membrane B) An enzyme complex in the cytoplasm. C) Somewhere within the cytoskeleton D) In the plasma membrane E) Prokaryotes do not perform cellular respiration.

D) In the plasma membrane

Dopamine is a neurotransmitter that functions as part of the reward-motivation behavior pathway. Which of the following is NOT true about this chemical? A) Drug addiction can cause a quick surplus of dopamine in your brain leading to a short lived, euphoric feeling. B) Its molecular structure includes a benzene (type of hydrocarbon) ring. C) Some people make too much and have to take medication to decrease their levels. D) It can be used as a vasoconstrictor (to narrow blood vessels) E) Some people make too little and have to take medication to increase their levels.

D) It can be used as a vasoconstrictor (to narrow blood cells) --> INCORRECT because it WIDENS vessels

Why is molecular oxygen (O2) so important for plant and animal life on earth? A) It oxidizes harmful macromolecule by-products of photosynthesis. B) It helps to remove carbon dioxide (CO2) from the atmosphere. C) It is the main building block for simple sugars. D) It is needed in the mitochondria in order to extract the greatest amount of ATP from glucose. E) It cleans the lungs out.

D) It is needed in the mitochondria in order to extract the greatest amount of ATP from glucose.

Which of the following is TRUE regarding chitin? A) Fungi are the only organisms that do not make use of chitin. B) Chitin is a structural polysaccharide not only in animals, but also in plants. C) The chitin monomer is in the "a" configuration. D) It is similar to cellulose except for the group shown by the arrow in the chitin monomer structure.

D) It is similar to cellulose except for the group shown by the arrow in the chitin monomer structure.

As electrons pass through the system of electron carriers associated with photosystem II, they lose energy. What happens to this energy? A) It is used for phosphorylation of NAD+ to NADPH, while NADPH accepts electrons from the photosystem I. B) It is lost as heat. C) It excites electrons of the reaction center of photosystem I. D) It is used to establish and maintain a proton gradient for the generation of ATP.

D) It is used to establish and maintain a proton gradient for the generation of ATP.

When an enzyme speeds up metabolic reactions, A) It increases delta G of the overall reaction. B) It is consumed by the reaction and cannot be reused for other processes. C) It lowers the delta G of the overall reaction. D) It lowers the activation energy necessary to break the bonds of the reactants.

D) It lowers the activation energy necessary to break the bonds of the reactants.

Which of the following DOES NOT support the fluid mosaic model? A) Phospholipids move laterally within the plane of the membrane. B) The presence of cholesterol greatly influences membrane fluidity. C) Membrane is composed of various biomolecules with properties affected by temperatures changes. D) Membrane proteins from cells of two different organisms will never be found mixed together in a hybrid cell.

D) Membrane proteins from cells of two different organisms will never be found mixed together in a hybrid cell.

If heavy exercise produces lots of CO2 in the body, will the blood become more acid or more basic? A) Can't determine B) Neither C) More basic D) More acid

D) More acid

Which of the following pieces of evidence is LEAST consistent with the endosymbiosis theory for the origin of the mitochondria? A) Mitochondria have their own DNA, which is more similar to modern bacteria than to modern eukaryotes. B) Mitochondria have circular DNA. C) Mitochondria replicate separately from the rest of the cell by a process similar to binary fission. D) Most of the proteins required for mitochondria functioning are found in the cell's nuclear DNA.

D) Most of the proteins required for mitochondria functioning are found in the cell's nuclear DNA.

What is true about DNA? A) Breaking an A-T pair is harder than breaking a C-G pair. B) DNA is synthesized in the 3' to 5' direction. C) All DNA mutations are deleterious. D) Nearly every single organism alive use the same exact genetic code (A, C, G, T/U). E) Members of the same species (i.e. all spotted owls) have identical genomes.

D) Nearly every single organism alive use the same exact genetic code (A, C, G, T/U).

The Calvin cycle of photosynthesis: A) Occurs in the thylakoids and results in the production of ATP and NADPH B) Occurs in the stroma and results in the production of ATP and NADPH C) Occurs in the thylakoids and results in the production of carbohydrate (sugar) D) Occurs in the stroma and results in the production of carbohydrate (sugar) E) Occurs in the stoma and results in the production of carbohydrate (sugar)

D) Occurs in the stroma and results in the production of carbohydrate (sugar)

Which of the following is the best definition or description for "chromatid"? A) A bundle of ribosomes with DNA wrapped around it. B) A fiber composed of a DNA molecule and proteins making up chromosomes. C) The region where two bodies are held together after DNA replication. D) One of the two bodies held together after DNA replication, each having an exact copy of the DNA. E) A type of organelle that is found in the membrane of animal cells.

D) One of the two bodies held together after DNA replication, each having an exact copy of the DNA.

The final electron acceptor for cellular respiration is: A) ADP B) NAD+ C) FAD D) Oxygen E) ETC

D) Oxygen

How is plant cell cytokinesis different from animal cell cytokinesis? A) Plants have cell walls and because of this do not fully undergo cytokinesis. B) The contractile filaments found in plant cells are structures composed of carbohydrates; the cleavage furrow in animal cells is composed of contractile proteins. C) Plant cells divide after metaphase but before anaphase; animal cells divide after anaphase. D) Plant cells deposit vesicles containing cell wall building blocks on the metaphase plate; animal cells form a cleavage furrow. E) The structural proteins of plant cells separate the two cells; in animal cells, a cell membrane separates the two daughter cells.

D) Plant cells deposit vesicles containing cell wall building blocks on the metaphase plate; animal cells form a cleavage furrow.

Why is it that denaturation makes proteins function poorly or not at all? A) Proteins are broken down partially or completely into their monomers by denaturation, and the monomers are not as effective for function. B) Protein function depends on constant denaturation provided by the DNA, and denaturation removes that protection. C) Denaturation removes critical amino acids that determine protein function. D) Protein function depends on shape, and denaturation damages protein shape. E) Denaturation adds nonpolar groups that change the solubility of certain proteins.

D) Protein function depends on shape, and denaturation damages protein shape.

Which of the following is NOT a function of a lipid? A) Component of cellular membranes B) Long-term energy storage C) Insulation from the environment D) Store genetic information E) Building blocks for some hormone synthesis

D) Store genetic information

Alfred Wallace is best known for what? A) The first person to describe a cell and credited with the cell theory. B) Proposing the idea of the inheritance of acquired traits. C) The theory of endosymbiosis leading to the evolution of the mitochondria & chloroplast. D) Suggesting natural selection as a mechanism for evolution. E) Being the "father of taxonomy" and coming up with a system of organizations.

D) Suggesting natural selection as a mechanism for evolution.

A full-length protein should result in the following "message": THE CAT ATE PIE AND RAN AND RAN. Which of the following messages would result from one nonsynonymous codon substitution? A) THE CAT ATE PIE AND RAN AND RAN B) THE CAT ATE PIE AND RAN AND RAN AND RAN AND RAN C) THE CAT TEP IEA NDR ANA NDR AN D) THE DOG ATE PIE AND RAN AND RAN E) THE PIE ATE CAT AND RAN AND RAN

D) THE DOG ATE PIE AND RAN AND RAN.

A valid scientific hypothesis must be: A) Funded (monetarily supported) B) Unable to be changed (permanent) C) Testable and Provable D) Testable and Falsifiable E) Provable

D) Testable and Falsifiable

When Kendrick Lamar said, "...I got loyalty, got royalty inside my DNA" which theory of biology was he referring to? A) The Prokaryotic Cell B) The Theory of Evolution C) The Endosymbiosis Theory D) The Gene Theory E) The Big Bang Theory

D) The Gene Theory

Which of the following is an example of potential rather than kinetic energy? A) A crawling beetle foraging for food B) A baseball sailing over the outfield wall after a batter hits a homerun. C) Water rushing over Niagara Falls D) The energy in a gravitational field. E) Light flashes emitted by a firefly.

D) The energy in a gravitational field.

Suppose you wanted to test the effect of fertilizer on the growth of tomato plants. What is the dependent variable? A) The presence or absence of fertilizer B) The type of fertilizer used. C) The amount of time that the plant was allowed to grow. D) The growth, or height, of the plants. E) The amount of water applied to each plant.

D) The growth, or height, of the plants

A patient experienced great blood loss. In an attempt to quickly replenish body fluids, a volume of distilled water equal to the blood lost was introduced directly into a blood vein. What is the most likely result of this treatment? A) The red blood cells are unaffected because distilled water shares the same pH with the intracellular region of red blood cells. B) The red blood cells are in danger of shrinking because they were subjected to a hypertonic environment. C) The red blood cells should be fine because they were subjected to an isotonic environment. D) The red blood cells are in danger of bursting because they were subjected to a hypotonic environment.

D) The red blood cells are in danger of bursting because they were subjected to a hypotonic environment.

Competitive inhibition differs from non-competitive inhibition by: A) The inhibitor concentration required to reduce enzyme activity. B) The site that the inhibitor binds to the substrate. C) The size of the inhibitor that can fit into the active site. D) The site that the inhibitor binds to the enzyme.

D) The site that the inhibitor binds to the enzyme.

From left to right: human check cells, human nerve cells, human red blood cells. Why do these three cells, taken from the same human, have different functions? A) They have different numbers of genes B) They use a different genetic code C) They have different sets of genes D) They have differential gene expression patterns E) All of the above

D) They have differential gene expression patterns

In taxonomy, what is true about the hierarchal rank of "family"? A) One species can belong to more than one family (but no more than 3 families). B) Members of the same taxonomic family can have some species with a nucleus and some without. C) A family will usually include far fewer taxa (plural of taxon) than a genus. D) Two species in the same family must be in the same taxonomic "Order". E) Two species in the same family must be in the same taxonomic "Genus".

D) Two species in the same family must be in the same taxonomic "Order".

Simple diffusion is "driven" by ________, while active transport works because of the use of _______? A) osmosis; ATP B) a concentration gradient; protein (channel or carrier); C) ATP; a concentration gradient D) a concentration gradient; ATP E) proteins (channel or carrier); a concentration gradient

D) a concentration gradient; ATP

If translation was successful the code should read: big boi fat cat ate the ham. Which of the following would result from a nonsense substitution? A) big boi fat cat ate the ham big boi fat cat ate the ham big boi fat cat ate the ham B) the ham ate the big boi fat cat C) bib oif atc ata tet heh am- D) big boi fat E) big boi fat rat ate the hem

D) big boi fat

Hydrangea plants of the same genotype are planted in a large flower garden. Some of the plants produce blue flowers and others pink flowers. This can be best explained by which of the following? A) the alleles are codominant B) a random mutation occurred on the gene causing pigmentation C) the knowledge that multiple alleles are involved D) environmental factors such as soil pH affect the phenotype E) the allele for blue hydrangea is completely dominant over the allele for pink hydrangea

D) environmental factors such as soil pH affect the phenotype

Gene S controls the sharpness of spines in a type of cactus. Cacti with the dominant allele, S, have sharp spines, whereas homozygous recessive ss cacti have dull spines. At the same time, a second gene, N, determines whether or not cacti have spines. Homozygous recessive nn cacti have no spines at all, therefore the S gene is not expressed one way or the other. The relationship between genes S and N is an example of which of the following inheritance patterns? A) codominance B) pleiotropy C) polygenic D) epistasis E) incomplete dominance

D) epistasis

Histone acetylation _______ transcription; while histone methylation _______ transcription. A) reduces; promotes B) turns off; turns on C) damages; repairs D) promotes; reduces E) repairs; damages

D) promotes; reduces

Acetylation of histones in the area of a gene causes that gene to A) turn off or down in expression levels. B) mutate and cause a change in gene functionality. C) alternatively splice during transcript processing. D) turn on or up in expression levels. E) amend damaged pieces via excision repair.

D) turn on or up in expression levels.

A particular viral genome can replicate in 8 hours. If you start with 55,000 copies of this genome, about how many copies would be present in a host's cell after 48 hours of growth? A) ~110,000 B) ~330,000 C) ~1,760,000 D) ~3,520,000 E) Correct amount not listed

D) ~3,520,000

A woman is found to have 47 chromosomes, including three X chromosomes. Which of the following statements describes her expected phenotype? A. A sterile female. B. A male who is sterile with feminine characteristics. C. A female with masculine characteristics (such as excess facial hair). D. A healthy female with slightly above average height. E. A female with severe multi‐system issues (i.e. cardiovascular, GI, renal).

D. A healthy female with slightly above average height.

When Mendel cross‐fertilized two true breeding pea plant varieties with alternate characteristics (such as green seed vs. yellow seed), the resulting offspring were: A. sterile. B. highly variable in phenotype. C. homozygous. D. the F1 generation. E. of intermediate phenotype.

D. The F1 generation

Which of the following is NOT a function or characteristic of a functional group? A) They can distinguish one group of biomolecules from another. B) They have consistent behavior from one molecule to another. C) They are part of a molecule most commonly involved in chemical reactions. D) They usually cause polymers to breakdown into monomers. E) They increase the solubility of organic compounds in water.

D. They usually cause polymers to breakdown into monomers.

Some characteristics appear with a continuous spectrum of variation within a population. For example, height in humans varies from very tall to very short with many intermediates. This inherited variation is best explained by: A. environmental influences. B. varying degrees of codominance. C. simple Mendelian inheritance. D. the presence of many genes acting jointly to produce the one phenotype. E. pleiotropic effects.

D. the presence of many genes acting jointly to produce the one phenotype.

During meiosis, when is DNA replicated?

DNA is replicated before meiosis I but NOT before meiosis II

Who explained the first mechanism for evolution, and what is it called?

Darwin & Wallace; natural selection

What is natural selection?

Differential reproductive success

What is the best definition for natural selection?

Differential reproductive sucess

Oxidative phosphorylation is the final part of cellular respiration. List the following steps of oxidative phosphorylation in sequential order (first to last according to what is provided) 1. H+ ions flow down a gradient to generate ATP. 2. Electrons are transferred from the CAC into the ETC. 3. Electrons are transferred to O2, causing it to split and take up H+ ions, which forms water. 4. H+ ions are pumped across the mitochondrial membrane to establish an electrochemical gradient. A) 2, 3, 1, 4 B) 4, 3, 2, 1 C) 1, 4, 3, 2 D) 1, 2, 3, 4 E) 2, 4, 3, 1

E) 2, 4, 3, 1

Quaking aspen trees can send out underground stems for asexual reproduction. Sexual reproduction is not as common, but when it does happen, the haploid gametes have 19 chromosomes. How many chromosomes are in the cells of the underground stems? A) 9.5 B) 76 C) 9 D) 19 E) 38

E) 38

The cells of a plant associated with leaf tissue development have a doubling time of 18 hours. If you start with 225,000 of these cells, approximately how many leaf cells would be present after 270 hours of growth? A) 3.4 million B) 61 million C) 330 million D) 225,000 E) 7.4 billion

E) 7.4 billion

A unicellular eukaryote undergoes multiple rounds of glycolysis and yields 48 molecules of pyruvate. How many total molecules of ATP will the investment payoff phase yield? A) 0 B) 192 C) 24 D) 48 E) 96

E) 96

Why is the one gene-one enzyme hypothesis is not entirely accurate? A) Some genes encode proteins that are not enzymes. B) Some genes encode a piece of a protein, not the entire protein. C) Some genes encode functional RNA molecules, not polypeptides. D) A and B only. E) A, B, and C.

E) A, B, and C

One codon for tyrosine (tyr) is UAU. The corresponding template DNA for this particular codon of tyr is ________ and the anticodon used during translation is _________. A) TAT, AUA B) TAT, ATA C) ATA, UAU D) AUA, ATA E) ATA, AUA

E) ATA, AUA

Based on a variety of experiments, it is known that the rate of photosynthesis is highest when wavelengths of light between 400 and 500 nm are used, and when wavelengths around 700 nm are used. What does this indicate about the relative importance of chlorophyll a and b in photosynthesis? A) Transmission of light by chlorophyll a and b is essential for the process of photosynthesis. B) Reflection of light by chlorophyll a and b is essential for the process of photosynthesis. C) Chlorophyll a and b are much less important than other pigments in the reflection, absorption, and transmission of light relevant to photosynthesis. D) Nothing. We cannot make any interferences with this information provided. E) Absorption of light by chlorophyll a and b is essential for the process of photosynthesis.

E) Absorption of light by chlorophyll a and b is essential for the process of photosynthesis.

Which statement about water's polarity is TRUE? A) Water's polarity causes other polar molecules to be hydrophilic. B) All are false. C) Water's polarity makes it a versatile solvent. D) Water molecules can form up to 4 hydrogen bonds with other water molecules. E) All are true.

E) All are true

The Cell Theory is one of the 4 major theories in biology. Which is correct? A) All cells have a nucleus. B) All organisms are made up of many cells. C) Chemical reactions necessary for life occur outside of the cells and are transported in through membrane interactions. D) Only certain cells (like neurons) contain hereditary information. E) All cells come from preexisting cells.

E) All cells come from preexisting cells.

Which of the following organisms perform anaerobic respiration? A) Humans B) Protists C) Fungi D) Bacteria E) All of the above

E) All of the above Explain: All because they all have cytoplasms which is where anaerobic respirations.

Which of the following is true about HeLa cells? A) They are an immortal line of cells that arose from a mutation. B) They have been used to develop many vaccines. C) They have been used to study and make major developments in HIV treatment. D) They were acquired controversially and lead to major changes/laws for patients and their healthcare rights. E) All of the above are true. Please read more about Henrietta Lacks.

E) All of the above are true. Please read more about Henrietta Lacks.

Nearly every organism functions on a 24 hour cycle. What does this circadian clock control/do? A) Affect sleep patterns B) Affect feeding patterns C) Regulate a bunch of stuff D) Determine when certain medicine should be taken to heighten their affect. E) All of the above.

E) All of the above.

Which of these is NOT an essential element that makes up living matter? A) Hydrogen B) Carbon C) Calcium D) Nitrogen E) All of these are essential elements. F) Oxygen G) Iron

E) All of these are essential elements.

Which of the following is a major difference between PS I and PS II of the light dependent reactions of photosynthesis? A) PSI gets an electron directly from the ETC that results after PSII. B) PSII gets an electron directly from the ETC that results after PSI. C) Light absorbance in PSI is most efficient at 700nm while in PSII it is more efficient at 680nm. D) Light absorbance in PSII is most efficient at 700nm while in PSI it is more efficient at 680nm. E) Both A and C are correct. F) Both B and D are correct. G) None of the above are correct. These two photosystems are identical.

E) Both A and C are correct.

A population of plants is growing in a nice, moist rain forest. All of the plants have equally thin, broad leaves that lose water quickly (which is idea in this environment). Which of the following could add a new heritable variation providing thicker, tougher leaves that would make the plants more drought-resistant? A) Gene flow: pollen from a nearby population of plants with thicker leaves could fertilize a flower and bring a new allele into the gene pool. B) Natural selection: survival of the fittest could create a new genetic adaptation like thicker leaves if it was needed. C) Environmental change: a drought could stress the plants, making them grow thicker leaves that they would pass to their offspring in order to survive. D) Mutation: a change to DNA in a sperm or egg could produce a new gene that controls leaf thickness. E) Both mutations or gene flow could allow this to happen.

E) Both mutations or gene flow could allow this to happen.

During the first stage in the light-independent reactions (or Calvin cycle) of photosysthesis, carbon dioxide is said to be "fixed". What does this mean? A) CO2 is used first as an electron donor, then an electron acceptor. B) CO2 is converted into RuBisCO. C) CO2 is released from the plant as a waste product. D) CO2 is converted from an organic compound to an inorganic compound. E) CO2 is converted from an inorganic compound to an organic compound.

E) CO2 is converted from an inorganic compound to an organic compound.

Earth's oceans are immense. Small floating plants called phytoplankton contribute to ocean productivity. As overall ocean productivity goes up, what would you predict would happen to global carbon dioxide levels? A) CO2 levels should remain the same. B) CO2 levels should increase. C) CO2 levels should vary depending on which ocean (location). D) CO2 levels should vary depending on the salinity of the ocean (salt concentration). E) CO2 levels should decrease.

E) CO2 levels should decrease.

Which of the following statements correctly describes the structure or function of chromatin? A) Euchromatin is not transcribed, whereas heterochromatin is transcribed. B) Both heterochromatin and euchromatin are found in the cytoplasm. C) Euchromatin is usually methylated whereas heterochromatin is acetylated. D) Heterochromatin is composed of DNA; euchromatin is made of DNA and RNA. E) Heterochromatin is highly condensed, whereas euchromatin is less compact.

E) Heterochromatin is highly condensed, whereas euchromatin is less compact.

A scientist finds a new protein in a rare Arctic fruit. Which of the following is the BEST prediction about the properties of this protein? A) It will be less than or equal to twenty protein monomers long. B) It will certainly be used by the plant to confer cold tolerance. C) It will likely have new protein monomers that have never been found in nature before. D) It most likely functions to maintain the fruit's structure. E) It will certainly have no more than 20 different types of monomers in its structures.

E) It will certainly have no more than 20 different types of monomers in its structures.

Tay-Sachs disease is a human genetic abnormality that results in cells accumulating and becoming clogged with very large, complex, undigested lipids. Which cellular organelle must be involved in this condition? A) Mitochondria B) Nucleus C) Golgi apparatus D) Endoplasmic reticulum E) Lysosomes

E) Lysosomes

Which of the following statements is a valid distinction between heterotrophs and autotrophs? A) Heterotrophs are the producers of the biosphere. B) Only heterotrophs require chemical compounds from the environment. C) Chloroplasts are only found in plants. D) Mitochondria are only found in heterotrophs. E) Only autotrophs can feed themselves using CO2 and other inorganic compounds.

E) Only autotrophs can feed themselves using CO2 and other inorganic compounds.

Which statement is correct? A) Water is a polymer. B) Polymers and monomers are basically the same thing. C) Monomers are made of polymers. D) Water is a monomer. E) Polymers are made of monomers.

E) Polymers are made of monomers. Explain: If we connect many monomers together, then we have a polymer.

The scientific method has great flexibility in the order of operations. Which of these is NOT a step in the process? A) Observation B) Communication C) Collecting data D) Repeating the experiment E) Proving your hypothesis

E) Proving your hypothesis --> You cannot prove a hypothesis -- it can either be supported or rejected.

Which of the following properties or processes do we associate with living things? A) Natural selection B) Creating energy C) Nucleated cells D) Sexual reproduction E) Responding to the environment

E) Responding to the environment

One codon for serine is UCG. The corresponding non-template DNA for this particular codon of serine is ____ and the anticodon used during translation is ____. A) TCG, TCG B) AGC, TCG C) AGC, AGC D) AGC, UCG E) TCG, AGC

E) TCG, AGC

Telomere shortening puts a limit on the number of times a cell can divide. Research has shown that telomerase can extend the life span of cultured human cells. How might adding telomerase affect cellular aging? A) Telomerase will speed up the rate of cell proliferation. B) Telomerase would have no effect on cellular aging. C) Telomerase would shorten the life span of the chromosome. D) Telomerase shortens telomeres, which delays cellular aging. E) Telomerase greatly reduces telomere shortening and slow aging.

E) Telomerase greatly reduces telomere shortening and slow aging.

In a healthy eukaryotic cell, the rate of DNA repair is typically equal to the rate of DNA mutation. When the rate of repair lags behind the rate of mutation, what is the most likely possible fate of the cell? A) DNA replication will continue by a new mechanism. B) RNA may be used instead of DNA as inheritance material. C) DNA replication will proceed more quickly. D) DNA replication will now proceed in the 3' - 5' direction to counteract this. E) The cell may be mutated into a cancerous cell.

E) The cell may be mutated into a cancerous cell.

The fact that a substrate must fit structurally and bind chemically to the active site of any certain enzyme explains why A) Enzymes slow down the speed of reactions. B) Enzymes are quickly used up during reactions. C) Any certain enzyme can act on only one very specific substrate. D) Enzyme activity is inhibited when the substrate binds. E) There are few enzyme-dependent reactions in cells.

E) There are few enzyme-dependent reactions in cells.

Which statement most accurately describes how the bulk of the carbohydrates you ate today are functioning in your body right now? A) They are being converted into a different form of energy and/or being stored for later use. B) They are serving as the primary structural component of the plasma membrane. C) They are synthesizing cellulose to help reinforce your cell wall structure. D) They are storing and replicating hereditary information. E) They are catalyzing a variety of chemical reactions.

E) They are catalyzing a variety of chemical reactions.

Starch and glycogen are important for the following reason: A) They are part of plasma membrane structure. B) They aid in DNA replication. C) They are signal molecules (hormones) D) They provide structural support. E) They are energy reserves for cells.

E) They are energy reserves for cells.

Which statement about enantiomers is FALSE? A) Their multiple forms are mirror images. B) All enantiomer forms have the same chemical formula. C) All statements are true for enantiomers. D) Their placement of atoms differ by having an asymmetric carbon. E) They differ in the arrangement of their covalent bonds.

E) They differ in the arrangement of their covalent bonds. Explanation: Enantiomers are isomers, have the same chemical formula, and are mirror images of one another. They cannot be superimposed.

Which of the following is NOT true about hydrocarbons? A) They have hydrophobic regions. B) They are only composed of hydrogens and carbons (but can make bonds with other compounds). C) They are a major source of combustible fuels. D) They are absent (without other bonds) in most living organisms. E) They easily dissolve in water.

E) They easily dissolve in water.

Why can certain types of viruses promote cancer in humans especially? A) The medicine used to treat the virus is what causes most cancers. B) The body overcompensates for damaged organs and mass produces cells. C) The viruses can cause silent mutations in the host's genome. D) The virus can mutate an oncogene into a proto-oncogene. E) They insert their DNA into the host genome and can cause massive damage.

E) They insert their DNA into the host genome and can cause massive damage.

What role do cofactors play, specifically coenzymes, in enzymatic reactions? A) They function to change the free energy released from the reaction. B) They function as allosteric regulators via the process of cooperativity. C) They temporarily bind to a substrate to change its shape or configuration to assist a reaction. D) They are usually lipid-based enzymes that regulate reactions. E) They temporarily bind to an enzyme to change its shape or configuration to assist a reaction.

E) They temporarily bind to an enzyme to change its shape or configuration to assist a reaction.

If LaMarck was correct in his thinking about the inheritance of aquired characteristics (or traits), which of the following would you be able to pass along to your kids? A) Your natural green eyes B) Your above average height C) Your blood type. D) Your inability to see colors. E) Your lack of an appendix after you had surgery to remove yours.

E) Your lack of an appendix after you had surgery to remove yours. Explanation: A-D are all heritable traits. LaMarck states, "If an organism changes during life to adapt to its environment, those changes are passed on to his offspring."

Vinblastine is a compound found naturally in the periwinkle flower and is a standard chemotherapeutic drug used to treat cancer. Because it interferes with the assembly of microtubules, its effectiveness must be related to A) suppression of cyclin production. B) acceleration of DNA synthesis. C) inhibition of cleavage furrow formation. D) blocking chromosome condensation. E) disruption of mitotic spindle formation.

E) disruption of mitotic spindle formation.

When placed in a 10% sucrose solution, red blood cells quickly shrivel (shrink and wrinkle up). When first placed in the solution, the interior of the cells is ____________ to the sucrose solution. After the cells shrivel, if equilibrium is reached, the cells' interior is __________ to the sucrose solution. A) isotonic, hypotonic B) hypertonic, isotonic C) hypertonic, hypotonic D) hypotonic, hypertonic E) hypotonic, isotonic

E) hypotonic, isotonic

In humans and fruit flies we can observe some sex‐linked recessive phenotypes that appear more frequently in males than females. This occurs because: A. dominant alleles are always present on the Y chromosome. B. the presence of sex‐linked genes on the X‐chromosome results in maleness. C. males are generally weaker and express diseases more readily. D. fathers cannot pass the sex‐linked alleles to their daughters. E. the gene on the X chromosome has no corresponding allele on the Y‐chromosome.

E. the gene on the X chromosome has no corresponding allele on the Y‐chromosome.

What is spermatogenesis?

Generation of sperm cells

When does the miotic spindle start to form?

In the cytoplasm during prophase

What is sister taxa?

Indicates lineages that share a branch point

What is a branch point?

Indicates where 2 lineages diverged

Where is eukaryotic DNA housed?

Inside the nucleus

Why is the miotic spindle needed?

It is needed to divide chromosomes in a parental cell into two daughter cells

A spotted lizard with a long tail is mated to a striped lizard with a short tail. Their offspring are all spotted with long tails. What is the probability of getting stripped lizards with long tails in the F2 generation?

P = SSLL x SSll F = SsLl x SsLl F2 = - 9 spots/long - 3 spots/long - 3 stripe/long - 1 stripe/short = 3/16

What is the phenotype of a homozygous dominant?

Purple

You are testing the effect of pH on coral bleaching. What is your INDEPENDENT variable in this experiment? A) The amount of coral bleaching. B) The type of coral used. C) The amount of time that the coral was allowed to grow. D) The pH of the water used. E) The presence or absence of water.

The pH of the water used.

What is the genotype of a homozygous recessive?

aa


Conjuntos de estudio relacionados

American Government Chapters warm ups

View Set

4/10-4/13 Words of Animals (Avenue)

View Set

Vocabulary Workshop Level F, Unit 1

View Set

Environmental Geology135 - Lecture Exam 1: Rocks

View Set